Criminal Law

Réussis tes devoirs et examens dès maintenant avec Quizwiz!

Which of the following statements is true about the effect at trial of the Miranda rules? A A confession obtained in violation of Miranda may be used to impeach the defendant's testimony. B A question at trial about the defendant's silence during an interrogation results in a mistrial. C A defendant's silence after receiving Miranda warnings may be used to counter an insanity defense. D If the police intentionally fail to give Miranda warnings, any statement obtained directly from the violation cannot be used at trial, but other evidence derived from such statements may be used.

A. A confession obtained in violation of Miranda may be used to impeach the defendant's testimony if the defendant takes the stand at trial and the confession was otherwise voluntary. However, the confession is not admissible in the state's case in chief as evidence of guilt, and a truly involuntary confession (e.g., one obtained through torture) is never admissible for any purpose. A question at trial about the defendant's silence during an interrogation might NOT result in a mistrial. A mistrial can be avoided if the admission was a harmless error. A single question may be harmless error if it is followed by an objection from the defense, sustained by the judge, and then the jury is instructed to disregard the question. A defendant's silence after receiving Miranda warnings may NOT be used to counter an insanity defense. (The prosecution's claim would be, how insane could the defendant have been? He understood well enough to remain silent after being told that anything he says can be used against him in court.) The warnings carry an implicit assurance that silence will carry no penalty. If the police intentionally fail to give Miranda warnings, the fruit of an interrogation may NOT be used at trial. Following an intentional failure by the police to give a detainee his Miranda warnings, if he gives the police information that leads to nontestimonial evidence, it will be suppressed. If the failure to warn was not intentional, however, the evidence will probably not be suppressed.

Under the Model Penal Code, consciously disregarding a substantial and unjustifiable risk indicates a person acted: A recklessly B knowingly C purposefully D negligently

A. A person acts recklessly when he consciously disregards a substantial and unjustifiable risk that circumstances exist or that a prohibited result will follow, and this disregard constitutes a gross deviation from the standard of care that a reasonable person would exercise in the situation. Recklessness is one of the four categories into which the Model Penal Code classifies the mental component of a criminal offense (i.e., the element of fault). A person acts purposefully when it is his conscious object to engage in certain conduct or cause a certain result. A person acts knowingly when he is aware that his conduct is of that nature or that certain circumstances exist. A person acts negligently when he fails to be aware of a substantial and unjustifiable risk that circumstances exist or a result will follow, and such failure constitutes a substantial deviation from the standard of care that a reasonable person would exercise under the circumstances.

A witness subpoenaed to testify before a grand jury has no right to: A Counsel in the courtroom, Miranda warnings, or warnings that she is a potential defendant B Warnings that she is a potential defendant, but she must be provided counsel in the courtroom and Miranda warnings C Counsel in the courtroom or Miranda warnings, but she must be warned if she is a potential defendant D Counsel in the courtroom, but she must be provided Miranda warnings and warned if she is a potential defendant

A. A witness subpoenaed to testify before a grand jury has no right to receive Miranda warnings. A grand jury witness also has no right to have an attorney present, but she may consult with an attorney outside the grand jury room. A witness who is under investigation and may well become a defendant has no right to a warning that she is a "potential defendant" when called to testify before the grand jury.

A bartender diligently followed the procedure her employer set: She would ask every patron for identification regardless of how old (or young) the patron appeared to be. One day, after asking for identification, the bartender served alcohol to a minor. The identification that the minor gave to the bartender was actually issued by mistake by an appropriate state agency and appeared to show that the minor was of legal age. After another patron, an off-duty police officer, recognized the minor, the bartender was arrested for serving alcohol to a minor. In this jurisdiction, the highest state court has held that, under state law, strict liability is abolished and all crimes require a culpable mental state. What would be the best reason for finding the bartender not guilty? A She did not know that the minor was underage, and she relied on the identification card for proof of age. B She did not know that the minor was underage, and therefore never intended to serve alcohol to a minor. C She made a diligent effort to determine the minor's age. D She checked the minor's state-issued identification card, which showed that the minor was of age.

A. An honest and reasonable mistake as to a material element of the offense would negate criminal liability for all crimes except strict liability offenses. Thus, if the state had abolished strict liability crimes, the bartender's mistake would be a defense regardless of the mental state required for the crime of serving alcohol to a minor. (B) is not as good an answer as (A). The bartender's lack of intent to commit the crime of serving alcohol to a minor would negate criminal liability if the crime required a specific intent, thus requiring an actual intention to engage in the act of serving alcohol to a minor. The question does not indicate the mental state required for the crime of serving alcohol to a minor. Even though the state had abolished strict liability offenses, the state could punish the crime of serving alcohol to a minor with a "reckless" or "should have known" state of mind. If so, the bartender's lack of intent would not result in a not guilty verdict. (C) is not as good an answer as (A) because the fact that the bartender made a diligent effort to determine the age of the minor would be an important consideration in deciding whether she made an honest and reasonable mistake, but it would not in and of itself automatically negate liability. A similar analysis applies to (D). The fact that the bartender checked the identification card supplied by a state agency would be an important consideration in deciding the nature of her mistake, but it would not by itself negate liability, as the mistake must be both honest and reasonable. For example, if the bartender knew that the minor was not of age despite what the identification card showed, she would commit a crime by serving the minor alcohol.

For the purposes of felony murder, the felony is deemed to terminate when: A the defendant reaches a place of temporary safety B the defendant leaves the scene of the felony C the victim dies D the last act required to complete the felony is committed

A. Deaths caused while fleeing a felony may also give rise to felony murder liability. Such liability, however, is generally terminated when the defendant reaches a place of temporary safety. Although the death of the victim is obviously required for a charge of felony murder, the felony is deemed to continue as to a fleeing felon until the felon reaches a place of temporary safety. It is conceivable that the felon may kill other individuals and thus be liable for multiple counts of felony murder. The completion of the felony, and leaving the scene of the felony, do not terminate potential felony murder liability. The felon may also be liable for deaths resulting from flight from the felony.

The Wharton rule states that: A If the crime requires two or more parties to commit it, there can be no conspiracy to commit the crime unless more parties take part in the crime than are necessary for its commission B There can be no conviction for conspiracy if all parties with whom the defendant has conspired have been acquitted C A conspiracy requires two or more "guilty minds" D An overt act in furtherance of the conspiracy is required, but an act of mere preparation will suffice

A. If two or more people are necessary for the commission of the substantive offense (e.g., adultery, dueling, sale of contraband), the "Wharton rule" states that there is no crime of conspiracy unless more parties participate in the agreement than are necessary for the crime. Traditionally, there can be no conviction for conspiracy if all parties with whom the defendant has conspired have been acquitted. However, this is not a statement of the Wharton rule. At common law, a conspiracy requires two or more "guilty minds." (The Model Penal Code and the modern trend is to follow a "unilateral approach" that requires only one guilty mind.) This is not, however, a statement of the Wharton rule. At common law, an overt act in furtherance of the conspiracy is not required; however, this is not the Wharton rule.

For Fourth Amendment purposes, which of the following people is LEAST likely to be found to have a reasonable expectation of privacy in the place searched? A A person who came to the premises that were searched to buy illegal drugs. B A person who owns the premises that were searched but does not live there. C A person who lives in the premises that were searched but does not own it. D A person who was an overnight guest at the place searched.

A. The Supreme Court has held that a person who was on the premises that were searched to buy illegal drugs does not have a reasonable expectation of privacy in the premises. A claim that a search violated the Fourth Amendment can be raised only by a person who has a reasonable expectation of privacy in the place searched. The Supreme Court has held that a person has a reasonable expectation of privacy any time she owns the place that was searched or has a right to possession of it; whether or not she lives there would not affect this interest. The Court has also held that if the place searched is the person's home, then she has a reasonable expectation of privacy, regardless if she owned or had a right to possess it. Finally, the Supreme Court has held that an overnight guest at the place searched also has a reasonable expectation of privacy in the premises for Fourth Amendment purposes.

The police obtained a valid arrest warrant for a drug dealer. A reliable informant told the police that the drug dealer was staying at a friend's house until "the heat was off." Without having obtained a search warrant, the police went to the friend's house, knocked on the door, and asked the friend if the drug dealer was there. The friend replied that the drug dealer had been staying at the house for a few days but had left a few hours ago. The police pushed open the door and began searching for the drug dealer. They found him hiding in a closet along with two five-pound bricks of marijuana. They arrested both the drug dealer and the friend. Before his trial for possession of marijuana, the friend moved to suppress the marijuana found in the closet. Should the court grant the motion to suppress? A Yes, because a search warrant was required. B Yes, because the police may not execute an arrest warrant at the third party's home. C No, because the police had probable cause to believe that the drug dealer was staying at the friend's home. D No, because the police had a valid arrest warrant and the marijuana was found incident to the arrest.

A. The court should grant the motion to suppress because a search warrant was required. Absent exigent circumstances, the police executing an arrest warrant may not search for the subject of the warrant in the home of a third party without first obtaining a separate search warrant for the home. If the police do execute an arrest warrant at the home of a third party without obtaining a search warrant for the home, the arrest is still valid but evidence of any crime found in the home cannot be used against the owner of the home because it is the fruit of an unconstitutional search. Thus, (A) is correct and (D) is incorrect. (B) is incorrect because it is too broad. A person can be arrested at the home of a third party, but the police generally cannot enter the third party's home without consent unless they have a search warrant for the home. (C) is incorrect because, as discussed above, a search warrant is required absent exigent circumstances, which are not present in this case. Here, the probable cause established by the informant's disclosure would have enabled the police to obtain a search warrant.

The defendant and the victim got into a minor verbal altercation, concluding with the defendant lightly shoving the victim. The victim lost his balance and struck his head on the pavement, causing serious bodily injury. The defendant was charged with battery, which is defined in the jurisdiction as "purposely or knowingly causing serious bodily injury to another." Should the defendant be convicted of battery? response - incorrect A No, because the defendant did not know that the victim would be seriously injured. B No, because the defendant did not strike a serious blow to the victim. C Yes, because the defendant purposely shoved the victim. D Yes, because the victim suffered serious bodily injury.

A. The defendant should not be convicted of battery. Under the statute's fault standards, a defendant must have acted purposely (i.e., with conscious intent to cause the result) or knowingly (i.e., with knowledge that his conduct will necessarily or very likely cause the result) as to the harmful result. The apparent inference to be drawn from the facts is that the defendant did not consciously desire, nor contemplate to a practical certainty, the serious injury to the victim that actually occurred. Had the defendant intended to cause such severe harm, he no doubt would have dealt the victim a strong blow rather than simply giving the victim a light shove. Therefore, as to the nature of the result, the defendant did not act with "purpose" or "knowledge" as those terms are defined in the Model Penal Code and modern criminal codes. (B), while close, is not as good an answer as (A) because it does not address the state of mind issue in the problem. A light shove might be sufficient for a battery as defined under a different set of facts (e.g., if the defendant believes that the victim would fall down stairs with a light shove). (C) is incorrect because it addresses the act but not the result. As defined in this question, battery must not only be committed by a purposeful act, but also be done with a "purposeful" or "knowing" state of mind as to the result. (D) is incorrect for much of the same reason—the state of mind requirement also applies to the result, as discussed above. The injury must have been purposely or knowingly caused, and that concept is not contained within choice (D).

A man and a woman were traveling in the man's car when they were stopped by the police for running a red light. Before the police came up to the car, the man told the woman, "You owe me a favor. Keep this package for me," and gave the woman a small foil package. The woman put the package in her backpack, saying, "O.K., but don't tell me what's in it." Before the police even began to question the occupants, the man blurted out, "I'm clean, man, but she has a stash," pointing at the woman. The officers searched the backpack that the woman was holding and found the foil package, which contained heroin. The woman was arrested, but the man was not. Is the evidence found on the woman admissible? A Yes, under the automobile exception. B Yes, because due process imputes knowledge where there is willful ignorance. C No, because due process forbids granting of immunity to the more culpable defendant. D No, because the woman did not know that the package contained heroin.

A. The evidence is admissible because the search was valid. Even though the police have validly stopped an automobile, they cannot search the vehicle without meeting the requirements of one of the exceptions to the warrant requirement, such as the automobile exception (which requires probable cause) or consent. The automobile exception comes into play when the police have probable cause to believe that the vehicle contains evidence of a crime. Under the exception, the police may search anywhere in the vehicle in which the item for which they have cause to search may be hidden, including packages in the vehicle. The statement of the man to the police officers gave them probable cause to believe that the car contained evidence of a crime (i.e., that the woman had drugs somewhere in the car). Thus, the requirement for application of the automobile exception was present, providing validity for the warrantless search conducted by the police. Because the search was valid, the evidence found on the woman is admissible. Besides being an incorrect statement of law, (B) is incorrect because it focuses on the woman's knowledge of the contents of the package. Whether the woman knew that heroin (or some other illegal substance) was in the package is irrelevant to the admissibility of the heroin. Even assuming that the woman knew of the contents, the search would not be valid unless there was a ground for the warrantless search. (D) similarly links the woman's knowledge of the package's contents to the admissibility of the evidence. As noted above, the admissibility of the evidence is dependent on the validity of the search that produced the evidence, rather than on the knowledge of the defendant as to the existence of the evidence. (C) is incorrect for three reasons: First, due process does not prohibit granting of immunity to a more culpable defendant. Second, there is no indication that immunity was even granted here (immunity from prosecution may be granted to compel a witness to answer questions. The facts merely state that the man was not arrested; this does not necessarily mean that he was granted immunity). Third, the call of the question relates to the admissibility of the evidence, and a grant of immunity does not relate to the question of the admissibility of the evidence found on the woman; such admissibility is determined by the validity of the search of the woman by the officers.

With regard to the crime of robbery, which of the following statements is true? A The force or threats of force may be used to retain possession immediately after such possession has been accomplished. B If intimidation is used, a threat to destroy the victim's dwelling house is insufficient. C If threats of immediate death or serious physical injury are used, they must be threats only to the robbery victim. D The property must be taken from the victim's person.

A. The force or threats must be used either to gain possession of the property or to retain possession immediately after such possession has been accomplished. If force is used, it must be sufficient to overcome the victim's resistance. If threats of immediate death or serious physical injury are used, they must be threats to the victim, a member of her family, a relative, or a person in her presence at the time. The property must be taken from the victim's person or presence. "Presence" means some location reasonably close to the victim, but it need not be taken from the victim's person. Property in other rooms of the house in which the victim is located is in her "presence." A threat to do damage to property will not suffice—with the exception of a threat to destroy the victim's dwelling house.

In a criminal proceeding, the fruit of the poisonous tree doctrine provides that: A not only must illegally obtained evidence (i.e., the "tree") be excluded at trial, but also all evidence obtained or derived from exploitation of that evidence (i.e., the "fruit") B illegally obtained evidence (i.e. the "tree") must be excluded at trial, but evidence obtained or derived from exploitation of that evidence (i.e., the "fruit") is admissible C evidence that a person has performed bad acts in the past (i.e., the "tree") can be introduced at trial to show a proclivity for committing the present criminal act (i.e., the "fruit") D evidence that a person has performed bad acts in the past (i.e., the "tree") cannot be introduced at trial to show a proclivity for committing the present criminal act (i.e., the "fruit")

A. The fruit of the poisonous tree doctrine provides that not only must illegally obtained evidence be excluded at trial, but also all evidence obtained or derived from exploitation of that evidence. There are exceptions to the rule (e.g., when officers rely in good faith on a facially valid search warrant; when seeking to impeach trial testimony), and the rule generally applies only at criminal trials (i.e., it does not apply at civil proceedings or parole proceedings); neither does it apply to violations of agency rules or state laws. The choice indicating that evidence obtained or derived from exploitation of illegally obtained evidence is admissible is the opposite of the exclusionary rule. The choices regarding evidence that a person has performed bad acts in the past are based on an evidence law concept. Generally, evidence of prior bad acts cannot be used to imply that the defendant acted improperly in the case being tried due to concerns of relevance and undue prejudice and confusion. In any case, these choices do not reflect what is commonly called the "fruit of the poisonous tree" doctrine in the lexicon of criminal procedure.

A husband discovered his wife in bed with a neighbor. The neighbor ran out the back door. The husband screamed at his wife and vowed revenge. After consuming several drinks to build up his nerve and becoming intoxicated, the husband went to his friend's house and borrowed a gun, and then went to the neighbor's house. The neighbor had neglected to lock his front door, so the husband walked in. He found the neighbor trembling in the living room and pointed the gun at him. The neighbor immediately began apologizing and pleading for his life, but then suddenly he pulled a switchblade knife from his pocket. As the metal flashed, the husband fired a single shot at the neighbor, killing him. What is the most serious crime of which the husband can be convicted? A Murder. B Manslaughter, because the husband was still distraught over finding the neighbor in bed with his wife. C Manslaughter, because his intoxication prevented the husband from having the requisite intent for murder. D No homicide crime, because the neighbor was about to attack him with a knife.

A. The husband can be convicted of murder. Murder is the unlawful killing of another human being with malice aforethought, which may be (i) intent to kill, (ii) intent to inflict great bodily injury, (iii) reckless indifference to an unjustifiably high risk to human life, or (iv) intent to commit a felony. Intentional use of a deadly weapon authorizes a permissive inference of intent to kill. Here, the husband uttered statements of revenge, confronted the neighbor with a loaded gun, and intentionally shot him when he pulled out a knife—more than enough evidence for a jury to find that the husband had the malice aforethought necessary for murder. Furthermore, none of the issues raised in the other choices will suffice to excuse the killing or reduce it to voluntary manslaughter. (B) is incorrect because the husband will not be able to meet all four tests for establishing the provocation necessary to reduce a killing from murder to voluntary manslaughter. The husband would have to offer evidence that (i) a provocation existed that would arouse sudden and intense passion in the mind of an ordinary person such as to cause him to lose his self-control, (ii) the husband was in fact provoked and lost his self-control, (iii) there was not sufficient time between the provocation and the killing for the passions of a reasonable person to cool, and (iv) the husband in fact did not cool off between the provocation and the killing. The husband can easily establish the first two elements, because discovery of one's spouse in bed with another person is virtually always considered adequate provocation by common law courts. However, the time interval between the provocation and the killing was probably sufficient for a reasonable person to cool off, and the facts strongly suggest that the husband did in fact cool off—he consumed several drinks to build up his nerve and went to a friend's house to get a gun before confronting the neighbor. Thus, a jury would probably reject a claim of voluntary manslaughter here. (C) is incorrect because the husband's voluntary intoxication would not preclude a finding of intent for murder. Because the husband became intoxicated to build up his nerve to kill the neighbor, a court would probably find that his intent at the time he began drinking would apply to his later conduct. Furthermore, voluntary intoxication is no defense to crimes involving recklessness. The husband can still be liable for murder based on a state of mind of reckless indifference to human life—his conduct in becoming intoxicated and then confronting the neighbor with a loaded gun is sufficient to establish that state of mind. (D) is incorrect because the homicide will not be excused on self-defense grounds. A person may use deadly force in self-defense only if (i) he is without fault, (ii) he is confronted with unlawful force, and (iii) he is threatened with imminent death or great bodily harm. The husband is not without fault, however, because he initiated the assault and prompted the neighbor to pull the knife. His status as the aggressor deprives him of the right to use force in his own defense under these circumstances.

Felony murder generally requires that: A The killing be committed during the course of the felony, the felony must be independent of the killing, and the death must have been a foreseeable result of the felony B The killing be committed during the course of the felony, the felony must be independent of the killing, and the defendant must have been convicted of the underlying felony. C The killing be committed during the course of the felony, the death must have been a foreseeable result of the felony, and the defendant must have been convicted of the underlying felony D The death must have been a foreseeable result of the felony, the felony must be independent of the killing, and the defendant must have been convicted of the underlying felony

A. To convict a defendant of felony murder, the prosecution must prove, beyond a reasonable doubt, that the defendant committed a felony (i.e., he is factually guilty of the felony). However, the defendant need not actually be convicted of the underlying felony if the statute of limitations for the felony has expired. The killing must take place while the felony is being committed. When the defendant reaches "a place of temporary safety," the felony is deemed terminated. The felony must be independent of the killing (e.g., the felony of manslaughter cannot be the underlying felony for felony murder). Finally, most states require that the death must be a foreseeable result of the commission of the felony.

A conspirator can be convicted of a crime committed by another conspirator if: response - incorrect A The crimes were committed in furtherance of the objectives of the conspiracy. B The crimes were committed in furtherance of the objectives of the conspiracy, and the crimes were foreseeable. C The crimes were committed in furtherance of the objectives of the conspiracy, or the crimes were foreseeable. D The crimes were foreseeable.

B. A conspirator can be convicted of a crime committed by another conspirator if the crimes were committed in furtherance of the objectives of the conspiracy AND the crimes were foreseeable.

An ineffective assistance of counsel claimant must show deficient performance and: A that counsel was not acting pro bono B that the result of the trial would have been different but for the deficient performance C that the deficient performance led to an adverse ruling by the judge D nothing more

B. An effective assistance of counsel claimant must show deficient performance and that the result would have been different but for the deficient performance (e.g., the defendant would not have been convicted or the sentence would have been shorter). An adverse ruling by the judge without actual prejudice to the defendant would not be sufficient to show ineffective assistance of counsel. The fact that the defense attorney may or may not have been acting pro bono (i.e., without payment) is irrelevant.

Miranda warnings are required as a prerequisite to the admissibility of confessions resulting from custodial police interrogation. Which of the following statements is true about the custody requirement under Miranda? A In considering whether a person was in custody during an interrogation, a court will consider the person's experience with the criminal justice system. B For Miranda purposes, a person may be in custody in his own home. C For Miranda purposes, a voluntary detention may constitute custody. D Whether a person is in custody depends on the subjective views of the person and the interrogator.

B. For Miranda purposes, a person may be in custody in his own home. Whether a person is in custody depends on whether his freedom of action is denied in a significant way. This is an objective test—whether a reasonable person under the circumstances would think that he is in custody. For example, if police officers handcuff a suspect in his own home, he is clearly in custody. Similarly, if they surround a person in the middle of the night in his own bedroom, awaken him, and begin questioning him, custody will be found. It is not true that custody depends on the subjective views of the person and the interrogator. As discussed above, the test is objective—what a reasonable person would think. Similarly, the courts will not consider a person's experience with the criminal justice system in determining custody. This is a subjective factor. Neither is it true that a voluntary detention may constitute custody. If a person consented to the detention, he has not been denied the requisite freedom of action; he is there by choice.

In which of the following situations does an initial aggressor have the right to use self-defense? A If the victim of the initial aggression fails to withdraw when he could safely do so. B If the victim of the initial aggression escalates a minor altercation into a deadly altercation. C If the initial aggressor becomes injured in the altercation. D If the victim uses force to defend himself.

B. If the victim of the initial aggression escalates a minor altercation into a deadly altercation, the aggressor has the right to use self-defense. Note that the aggressor may also use self-defense if he, in good faith, effectively removes himself from the fight, and communicates to the other person his desire to remove himself. Being injured in the altercation does not give the aggressor the right to use self-defense. If the victim uses force to defend himself, and it does not escalate the fight into a deadly altercation, the aggressor does not have the right to use self-defense. A person confronted with nondeadly force may use such force as he reasonably believes is necessary to defend himself. There is no requirement to retreat before using nondeadly force, and one does not become "at fault" for failing to do so.

In most states, murder is of the second degree, but the murder is of the first degree when: A deliberation and premeditation can be shown, or the defendant acted "with a depraved heart" B deliberation and premeditation can be shown, or the killing occurred during an enumerated felony C "a depraved heart" can be shown D the killing occurred during an enumerated felony, or the defendant acted "with a depraved heart"

B. In most states, murder is of the second degree unless deliberation and premeditation can be shown, or the killing occurred during an enumerated felony, in which case the murder is of the first degree. If neither can be shown, the killing will usually be second degree murder (unless the killing is downgraded to manslaughter based on adequate provocation). Depraved heart murder would ordinarily be second degree murder in most states.

In which of the following situations is there sufficient intent to constitute larceny? A Taking goods with the belief that the taker is entitled to them as repayment for a debt of the owner. B Taking goods that are not for sale with the intent to pay the owner. C Taking goods with the intent to return them and receive a reward. D Taking goods with the intent to return them within a reasonable time.

B. Taking goods that are not for sale with the intent to pay the owner constitutes larceny. If the goods taken are not for sale, the fact that the defendant intends to pay the owner for them does not negate the larceny. (If the goods are for sale and the defendant has a specific and realistic intent to repay the owner, the taking is not larceny.) Taking goods with the intent to return them within a reasonable time does NOT constitute larceny if at the time of the taking the defendant has a substantial ability to do so. However, many states make it a crime to borrow a motor vehicle, even when the borrower fully intends to return it ("joyriding"). Taking goods with the honest belief that the taker is entitled to them as repayment for a debt of the owner is NOT larceny, but the goods must not be worth more than the amount of the debt. In these situations, the defendant believes the property is hers and therefore lacks an intent to deprive someone else of his property. Taking goods, intending to return them and receive a reward is NOT larceny . However, if the defendant takes them, not intending to return them unless she is assured of a reward, she has committed larceny, because she has created a substantial risk of loss.

A grand jury was investigating a bank robbery. The only information known to the prosecutor was a rumor that a certain ex-convict might have been involved. The grand jury subpoenaed the ex-convict. He refused to answer questions about the robbery and was granted use and derivative use immunity. He then testified that he and a friend had robbed the bank. The grand jury indicted both the ex-convict and his friend for the bank robbery. The prosecutor permitted the friend to enter a plea to a lesser offense in exchange for the friend's agreement to testify against the ex-convict. The prosecutor had no evidence as to the identity of the robbers except the testimony of the friend and the ex-convict. At the ex-convict's trial, should his objection to his friend's being permitted to testify be sustained? A Yes, because the prosecutor may not bargain away the rights of one co-defendant in a deal with another. B Yes, because the friend's testimony was acquired as a result of the ex-convict's grand jury testimony. C No, because the police suspected the ex-convict even before he testified in the grand jury hearing. D No, because a witness cannot be precluded from testifying if his testimony is given voluntarily.

B. Testimony obtained by a promise of immunity is by definition coerced and therefore involuntary. Thus, immunized testimony may not be used for impeachment of the defendant's testimony at trial. The friend's testimony will not be permitted to be used against the ex-convict because it resulted from the ex-convict's immunized testimony, and use and derivative use immunity prevents, as the term states, use of the immunized testimony and use of any evidence derived from the immunized testimony. (A) is wrong because it is an inaccurate statement of the law. Prosecutors can bargain away the rights of co-defendants. (C) is wrong because police suspicion is not the equivalent of actual testimony. (D) is wrong. Even though a witness wants to testify, various privileges such as lawyer-client, doctor-patient, etc., may bar the testimony. Here, the grant of immunity to the ex-convict is a bar to the friend's derived testimony because use immunity bars use of one's testimony or anything derived from it.

Which of the following statements is true regarding a detainee's Fifth Amendment right to counsel under Miranda? response - correct A All doubts about a request for counsel are construed in favor of the detainee. B The right applies only at custodial interrogations by the police or one known to be an agent of the police. C The police may question the detainee about an unrelated crime if they scrupulously honor the request by waiting a few hours and rewarning the defendant before the new questioning begins. D A detainee cannot waive the right to counsel in the absence of counsel.

B. The Fifth Amendment right to counsel under Miranda applies only to custodial interrogations by the police or one known to be an agent of the police. The purpose of the rule is to protect the right against self-incrimination by preventing the police from badgering a suspect until he talks. The warnings are intended to offset the coercive atmosphere of custodial police interrogation, and so the right does not apply when the suspect is not in custody or is not being questioned by a police officer or one known by the suspect to be an agent of the police. If a detainee invokes his Miranda right to counsel, the police may NOT question him about an unrelated crime. This is different from the rule for when the detainee merely invokes his right to remain silent, in which instance the police may question about a different crime if they scrupulously honor the request. It is not true that all doubts about a request for counsel are construed in favor of the detainee. A request for counsel must be unambiguous and specific. Neither is it true that a detainee cannot waive the right to counsel in the absence of counsel. A waiver will be valid if it was knowing, voluntary, and intelligent.

The probable cause to arrest standard requires that a police officer have __________ of reasonably trustworthy facts and circumstances sufficient to warrant a reasonably prudent person to __________ that the suspect has committed or is committing a crime for which arrest is authorized by law. A Personal knowledge; believe beyond a reasonable doubt B Knowledge; believe C Personal knowledge; believe D Knowledge; believe beyond a reasonable doubt

B. The best statement of the probable cause requirement is the officer must have knowledge (but not necessarily personal knowledge) of reasonably trustworthy facts and circumstances sufficient to warrant a reasonably prudent person to believe that the suspect has committed or is committing a crime for which arrest is authorized by law. This is a totality of the circumstances test, and a police officer's training and experience can come into play. The choices indicating that the officer must have personal knowledge of the facts are incorrect; the officer can base the decision on reliable "tips" from others, on a police report, on a police radio broadcast, or the like. The choices indicating that the officer must believe beyond reasonable doubt that the suspect is committing or has committed a crime are incorrect because the standard is too onerous; it is sufficient that a reasonable person would believe that a crime has been or was being committed.

Some constitutional rights applicable in criminal cases are offense specific (i.e., when invoked, they apply only to the specific charge for which they were invoked), and some are not offense specific (i.e., once invoked, they apply to all charges against the defendant and not just the charge for which they were invoked). Which of the following statements is correct regarding whether the Fifth and Sixth Amendments are offense specific? A Neither the Fifth Amendment nor the Sixth Amendment is offense specific. B The Fifth Amendment is not offense specific, but the Sixth Amendment is offense specific. C The Fifth Amendment is offense specific, but the Sixth Amendment is not offense specific. D Both the Fifth Amendment and the Sixth Amendment are offense specific.

B. The correct choice is that the Fifth Amendment is not offense specific, but the Sixth Amendment is offense specific. Thus, if a defendant invokes his right to counsel, under the Fifth Amendment the police cannot interrogate the defendant about any charge without counsel. But under the Sixth Amendment, the defendant can be interrogated regarding a different charge. The difference is significant when a defendant requests counsel after being charged and is put into a cell with an informer. The Sixth Amendment right to counsel applies to any post-charge interrogation—whether or not the defendant knows he is being interrogated by a government agent. The Fifth Amendment right to counsel, on the other hand, applies only when the defendant knows that he is being interrogated by a government agent. Thus, an undercover informer in the defendant's cell can question the defendant about anything without violating his Fifth Amendment right to counsel, and under the Sixth Amendment, the informer can question the defendant about any crime but the one with which he is charged. Therefore, if the cellmate informer questions the defendant and obtains information regarding a crime different from the one with which the defendant was charged, the questioning violates neither the Fifth nor the Sixth Amendment.

One night when a man was very drunk, he took one of his rifles, loaded it, and fired a bullet through his front door. Unbeknownst to him, at the time he fired the rifle, someone was driving by the house. The bullet went through the front door, through the window of the car, and killed the driver. The shooter was convicted of murder and appeals. He contends that there was insufficient evidence to support a finding of murder. How should the court of appeals rule? A That the evidence is sufficient to prove that the killing was intentional. B That the evidence is sufficient to prove that the killing was done with malice aforethought. C That the evidence is insufficient, because the shooter did not know that the driver was driving by his house and therefore he could not have acted intentionally. D That the evidence is insufficient, because at most the shooter's conduct constituted gross negligence and involuntary manslaughter.

B. The court of appeals should rule that the evidence is sufficient to prove that the killing was done with malice aforethought. Under the facts of this case, to support a finding of murder, the trial court would have to find that the shooter acted either intentionally or with malice aforethought. The facts clearly indicate that the shooter did not know of the car, so it cannot be said that he shot at it intentionally, and therefore (A) is not correct. "Malice aforethought" can mean that the defendant is acting with reckless indifference to an unjustifiably high risk to human life. There is little question that shooting a rifle through a front door shows reckless indifference to an unjustifiably high risk to human life. Thus, the question is whether the shooter's intoxication was sufficient to negate this state of mind. Voluntary intoxication is not a defense to crimes requiring malice, recklessness, or negligence. In the case of recklessness, if a defendant's lack of awareness results from voluntary intoxication, his conduct will nevertheless be deemed reckless. (C) is not a correct analysis of the issue, because his intentional act was firing the rifle, not shooting at the car. (D) is not the best answer, because although there is the possibility that the prosecution might have been able to show only gross negligence, there is sufficient evidence to support a finding of malice aforethought and murder.

A police officer learned from a reliable informant that a major drug deal was about to take place at a local restaurant. The officer obtained a search warrant for the restaurant and arrived with other uniformed officers to search the premises. While conducting the search, the officer searched several of the customers. While searching one of the restaurant's regular customers, the officer felt an object in the customer's pocket and pulled out a container filled with heroin. The customer was arrested and later convicted of possession of heroin. A state statute permits officers executing a search warrant to search persons on the premises if the officers reasonably expect danger to themselves or a risk of disposal or concealment of anything described in the warrant. If the customer challenges his conviction on the ground that his Fourth Amendment rights were violated, will he be successful? A Yes, because the statute is vague and overbroad. B Yes, because his presence in the place to be searched by the police does not negate the requirement of probable cause. C No, because the search was conducted pursuant to a valid search warrant. D No, because the search was authorized by statute.

B. The customer will be successful. To be reasonable under the Fourth Amendment, most searches must be pursuant to a warrant. The warrant must describe with reasonable precision the place to be searched and the items to be seized. A search warrant does not authorize the police to search persons found on the premises who are not named in the warrant. In Ybarra v. Illinois (1979), a case based on similar facts, the Supreme Court held that "each patron of the tavern had an individual right to be free of unreasonable searches, and presence at a location subject to search does not negate the requirement of probable cause to search the person present." (A) is incorrect because the validity of the statute is not the primary issue. Even in the absence of a statute, the search of the customer by the officer violated the customer's Fourth Amendment rights. (C) is incorrect because, as discussed above, the search warrant did not override the customer's Fourth Amendment rights. While the police would be able to search a person discovered on the premises for whom they had probable cause to arrest, because the search would be incident to a lawful arrest, here they searched the customer prior to an arrest and without probable cause. (D) is irrelevant; if a search is unconstitutional, it does not matter that it was authorized by statute. To the extent that the statute authorizes a search in violation of the Fourth Amendment, it is unconstitutional.

Suspecting criminal activity, a police officer acting without a warrant peeked through a small opening in the shutters of an apartment. The officer observed the apartment's tenant and the defendant making methamphetamine. The officer immediately entered the apartment and arrested the tenant and the defendant, and he confiscated the ingredients for the methamphetamine, the tools used for methamphetamine production, and any completed methamphetamine for evidence. The search is later ruled invalid at a suppression hearing. May the defendant now claim that her Fourth Amendment rights have been violated by the seizure of the ingredients, tools, and methamphetamine from the apartment? A Yes, because the items will be used in evidence against her. B Yes, if she was an overnight guest of the tenant. C No, because she was not the owner or occupier of the apartment. D No, unless she admits to ownership of the items.

B. The defendant can claim a reasonable expectation of privacy for Fourth Amendment purposes if she was an overnight guest of the owner of the place searched. To raise a Fourth Amendment claim of an unreasonable search or seizure, a person must have a reasonable expectation of privacy with respect to the place searched or the item seized. It is not enough merely that someone has an expectation of privacy in the place searched. The Supreme Court has imposed a standing requirement so that a person can complain about an evidentiary search or seizure only if it violates her own reasonable expectations of privacy. The Court has held that a person has a reasonable expectation of privacy any time (i) she owned or had a right to possession of the place searched, (ii) the place searched was in fact her own home, whether or not she owned or had a right to possession of it, or (iii) she was an overnight guest of the owner of the place searched. Thus, the defendant would have standing to challenge the search of the tenant's apartment if she was an overnight guest of the tenant. (A) is incorrect because standing to raise a Fourth Amendment claim does not exist merely because a person will be harmed by introduction of evidence seized during an illegal search of a third person's property. The person must establish that her own reasonable expectation of privacy has been violated. (C) is wrong because the fact that the defendant was not the owner or occupier of the apartment does not preclude her from challenging the search. As discussed above, an overnight guest may also have a reasonable expectation of privacy in the premises for purposes of the Fourth Amendment. (D) is incorrect. Although the defendant may have standing to object to the seizure of items if she claims ownership of them, that is not the only basis for raising a Fourth Amendment claim; she will have standing to object to the search of the apartment under the circumstances in (B) regardless of whether she claims ownership of the methamphetamine.

Two robbers planned to commit armed robberies targeting older victims. However, when the time came to actually commit the robbery, one of the robbers, thinking that the potential victim looked too much like his grandmother, backed out and told his cohort that he was going home. The second robber went ahead with the plan and robbed the elderly victim, who died of a heart attack due to the stress of the robbery. The second robber was arrested and implicated the first robber. What crime(s) has the first robber committed? A No crimes. B Conspiracy. C Murder. D Murder and conspiracy.

B. The first robber is guilty of conspiracy but not murder. Conspiracy consists of: (i) an agreement between two or more persons; (ii) an intent to enter into an agreement; and (iii) an intent to achieve the objective of the agreement. In addition, most states require an overt act in furtherance of the conspiracy, although an act of mere preparation will usually suffice. The conspiracy was complete when the robbers agreed to commit the robbery and targeted their first victim. The first robber's withdrawal is no defense to the conspiracy charge because a conspiracy is complete as soon as an agreement is made and an overt act is committed. Hence, (A) is incorrect. The first robber is not guilty of murder, however, because of his withdrawal. The murder charge would be based on felony murder, because the second robber caused the foreseeable death of the elderly victim from the heart attack during the commission of the felony. However, a conspirator may limit his liability for subsequent acts of other conspirators by performing an affirmative act that notifies the other members of the conspiracy in time for the other members to have the opportunity to abandon their plans. Here, the first robber told the second robber that he was going home in time for the second robber to abandon his plans. Hence, he is not liable for felony murder arising from the robbery, making (C) and (D) incorrect.

Which of the following describes sufficient intent to convict a defendant of an attempted crime? A The awareness by the defendant that he is acting in a proscribed way and that any attendant circumstances required by the crime are present. B The intent to perform an act and obtain a result that, if achieved, would constitute a crime. C A reckless disregard of an obvious or high risk that a particular harmful result might occur from the conduct. D The intent required for the completed offense.

B. The intent required for an attempted crime is the intent to perform an act and obtain a result that, if achieved, would constitute a crime. Note that regardless of the intent required for a completed offense, an attempt always requires a specific intent. The awareness by the defendant that he is acting in a proscribed way and that any attendant circumstances required by the crime are present more closely describes "general intent." Attempt is a specific intent crime. A reckless disregard of an obvious or high risk that a particular harmful result might occur from the conduct more closely describes "malice." Attempt is a specific intent crime.

Two robbers planned to rob a local convenience store, with one using a gun to force the clerk to turn over all of the money in the cash register while the other stood lookout near the door. The robbery did not go as planned. Instead of turning over any cash, the store clerk tried to disarm the gunman. During their struggle for the gun, the lookout decided that her best course of action was to grab what she could and flee the scene. The lookout took a newspaper and a bag of potato chips and ran out of the store. On her way out, she heard a gunshot. Later that day, she learned from news accounts that the gun accidentally discharged, killing the gunman. After an investigation, the lookout was arrested. If the lookout is charged with felony murder, what would be her most promising defense? A She did not intend for the gunman to get killed. B The only person killed was the gunman. C The killing occurred after the robbery was over. D The robbery was not a felony because the items that the lookout took had only minimal value.

B. The lookout's best defense is that the gunman was the only person killed. Under the felony murder doctrine, a killing committed during the course of a felony is murder, malice being implied from the intent to commit the underlying felony. However, under the majority view, criminal liability for murder cannot be based on the death of a co-felon from resistance by the victim or police pursuit. Thus, given that the gunman's death resulted from an act by the clerk, the victim of the robbery, the lookout cannot be found guilty of the felony murder of the gunman, a co-felon. (A) is incorrect because any desire or lack of desire by the lookout to see her co-felon harmed is irrelevant to liability for felony murder. The only mens rea required is the intent to commit the underlying felony. Here, the lookout had the intent to commit robbery, the underlying felony. From this intent, the malice required for murder is implied. (C) is incorrect because the fact that the felony was technically completed before the gunman's death does not prevent the killing from being felony murder. A death caused while fleeing from the crime is considered to have been caused during the commission of the felony. (D) is incorrect because robbery is a felony regardless of the value of the property that is taken.

The search incident to arrest exception to the warrant requirement applies: A Only if the police fear for their safety B After any constitutional arrest C After any arrest, even if the arrest itself is unconstitutional D Only if the arrest is for a felony crime

B. The police may conduct a search incident to arrest after any constitutional arrest. The Fourth Amendment prohibits unreasonable searches and seizures. Generally, for a search to be valid, it must be pursuant to a warrant issued by a neutral and detached magistrate and based on probable cause to believe that seizable evidence or fruits of a crime will be found on the premises to be searched. However, searches incident to arrest are an exception to the general rule. The choice that this search can be made after any arrest, even if it is unconstitutional is wrong. The police may conduct a search incident to an arrest whenever they arrest a person, even if the arrest is invalid under state law, as long as the arrest was constitutionally valid (e.g., reasonable and based on probable cause). Although the exception is based on safety concerns, the Supreme Court has held that it is not necessary that the police fear for their safety for a warrantless search to be conducted following an arrest. The search may be conducted incident to arrest in any case, as long as it is conducted contemporaneously with the arrest. It is not true that the exception applies only in the case of felony arrests. As discussed above, the exception applies after any lawful arrest.

A former construction worker became intoxicated one night and decided to move some heavy construction equipment that was parked at a construction site. Ignoring "no trespassing" signs, the worker jumped the fence and climbed into a large dump truck and started it up. However, even though he knew how to operate the truck, he quickly lost control of it due to his intoxication. It rumbled a short distance and crashed into a trailer housing the main office of the construction site. The worker is prosecuted for recklessly damaging property. A separate statute in the jurisdiction prohibited the unauthorized operation of construction equipment. Should the worker be found guilty of recklessly damaging property? A Yes, because his actions constituted an unlawful operation of the construction equipment. B Yes, because he was intoxicated while attempting to move the construction equipment. C No, because at most he could be found guilty of criminal negligence. D No, because he must have been aware that his conduct would cause the damage to the trailer in order to be found guilty of reckless damage.

B. The worker should be convicted because he was intoxicated when he damaged the trailer. The worker is being charged with reckless damage to property. A person acts recklessly when he consciously disregards a substantial and unjustifiable risk that a prohibited result will follow, and this disregard constitutes a gross deviation from the standard of reasonable care. Attempting to move a large piece of construction equipment while intoxicated should be considered reckless conduct because of the great potential for destruction arising from the size and destructive power of the construction equipment. Therefore, (B) is correct. (A) is incorrect because merely operating the equipment in violation of the law would not necessarily be reckless. For instance, here, the statute prohibiting unauthorized operation of the equipment likely was enacted to prevent untrained persons from driving dangerous equipment, but the worker was trained to operate the truck in question; thus, if not for the fact that he was drunk, his action would not necessarily have been reckless. Violating the statute may be evidence of negligence, but negligence is insufficient to establish recklessness. (C) is incorrect for the same reason that (B) is correct—driving the equipment while intoxicated constitutes reckless conduct. Although voluntary intoxication is a defense to a crime that requires purpose or knowledge, it is no defense to crimes involving recklessness. Even though the worker's condition may in fact have precluded him from being consciously aware of the risk, one who is not consciously aware of a risk only because he was voluntarily intoxicated will be deemed to have acted recklessly with regard to the risk. (D) is incorrect because it states the mental state for knowing conduct—if the defendant is aware that his conduct will necessarily or very likely cause a certain result, he acts knowingly with respect to that result. Recklessness is a lesser standard of fault.

The police of a resort town discovered that a well-known cat burglar was currently living in town under an assumed name. To try to catch her in the act of burglary, an undercover officer approached the burglar with a plan for a burglary. The undercover officer told the burglar that he knew who she was and that he had a plan to steal jewels from someone staying in one of the town's resorts. The burglar initially refused the offer; however, after lengthy cajoling, she finally agreed to the plan. As the time for the burglary drew near, the burglar had second thoughts. Three hours before the theft was scheduled to take place, the burglar called the police and told them of the plan. She told them that she was not going to show up, but that her cohort (the undercover officer) would be there, and told them how to recognize the undercover officer. Is the burglar guilty of conspiracy at common law? A Yes, because the burglar made an agreement with the undercover officer to commit the theft. B No, because there was no agreement. C No, because the intended crime was never completed. D No, because the burglar effectively withdrew.

B. There was an insufficient agreement for conspiracy liability at common law. Conspiracy consists of: (i) an agreement between two or more persons; (ii) an intent to enter into an agreement; and (iii) an intent to achieve the objective of the agreement. In addition, most states require an act in furtherance of the conspiracy, although an act of mere preparation will usually suffice. The agreement requirement means that the parties must agree to accomplish the same objective by mutual action. There must be a meeting of at least two "guilty minds"; i.e., between two or more persons who are actually committing themselves to the scheme. If one person in a two-party conspiracy is only feigning agreement, the other person cannot be convicted of conspiracy under the common law bilateral approach. Here, the officer, in his undercover capacity, was simply trying to set up a situation in which the burglar would be caught in the act. Thus, the undercover officer merely pretended to reach an agreement with the burglar to commit a burglary. At no time did the undercover officer actually commit himself to the burglary. Therefore, there could have been no agreement of two "guilty minds." Absent the requisite agreement, the burglar cannot be guilty of conspiracy. (A) is incorrect because, as explained above, there was no agreement sufficient for a conspiracy conviction, since the undercover officer never intended to commit the burglary. (C) is incorrect because completion of the substantive crime is not necessary for a conviction of conspiracy. Consequently, although the actual burglary was not consummated, this would not preclude a conviction of conspiracy to commit burglary. (D) is incorrect because withdrawal is not a defense to a charge of conspiracy. Note that, by withdrawing, a person may limit her liability for subsequent acts of the other members of the conspiracy. However, this question pertains to the burglar's potential guilt for conspiracy. As applied to the conspiracy charge, withdrawal will not afford a defense to the burglar.

Which of the following is most likely to be found to be a seizure for purposes of the Fourth Amendment? response - correct A A police officer approaches a suspect, orders the suspect to stop, and the suspect runs. B A police officer approaches a suspect and draws her revolver, orders the suspect to stop, and the suspect complies. C A police officer turns on his squad car's overhead lights as the squad approaches a suspect, and the suspect runs. D A police officer boards a bus, asks a suspect for identification and consent to search his luggage, and the suspect agrees.

B. f a police officer approaches a suspect and draws her revolver, orders the suspect to stop, and the suspect complies, this will most likely be found to constitute a seizure. For purposes of the Fourth Amendment, a seizure occurs when a reasonable person would believe that he is not free to leave. The courts consider the totality of the circumstances in making this determination. If a police officer turns on his squad car's overhead lights as the squad approaches a suspect and the suspect runs, a court will not likely find that there was a seizure. A seizure requires a physical application of force (e.g., handcuffing or otherwise subduing a person) or submission to an officer's show of force. If a police officer approaches a suspect, orders the suspect to stop, and the suspect runs, there is no application of force and no submission to the show of force, which would require, at the least, stopping in response to the officer's order. Therefore, it is unlikely that a court would find a seizure under the Fourth Amendment in these circumstances. If a police officer boards a bus, asks a suspect for identification and consent to search his luggage, and the suspect agrees, a court would not likely find a seizure for Fourth Amendment purposes. Police officers may ask people for permission to search and for identification; such requests do not involve the physical application of force or submission to a show of force.

A husband and wife were charged with stealing credit cards and charging expensive items on the misappropriated cards. An attorney was appointed by the court to represent the couple jointly. At the preliminary hearing, the judge found that the attorney would have no conflict representing both defendants in the joint trial. Halfway through the trial, however, a conflict arose between the defenses of the husband and wife. At the wife's request, the attorney moved that another attorney be appointed to represent the wife and that a mistrial be declared. The trial judge moved favorably on the attorney's motion. Another attorney was appointed to represent the wife, and as soon as the wife's trial began, her attorney moved to dismiss the case on the ground that jeopardy had attached during the wife's first trial and that she was being retried in violation of the United States Constitution. Should the judge grant the wife's attorney's motion? A Yes, because jeopardy attached when the jury began to hear evidence in the first trial. B Yes, because the judge incorrectly ruled that there would be no conflict of interest from the joint representation. C No, because the wife requested the mistrial. D No, because it is premature to move for a dismissal based on double jeopardy until the defendant is convicted.

C. Although jeopardy attached in the wife's first trial, her retrial is not barred because she initiated the grant of the mistrial in her first trial. As a general rule, the right to be free of double jeopardy for the same offense bars a retrial for the same offense once jeopardy has attached in the first trial. However, one of the exceptions permitting retrial even if jeopardy has attached is when a mistrial is granted in the first trial at the request of the defendant on any ground not constituting an acquittal on the merits. Here, the wife requested the mistrial because a conflict arose between the defenses of her and her co-defendant in the joint trial, and the judge granted the mistrial solely to allow the wife to obtain another attorney. Thus, no acquittal on the merits occurred and the double jeopardy rule does not apply. (A) is incorrect for several reasons. Merely because jeopardy attaches does not mean that the double jeopardy rule will apply; retrial will be permitted under certain exceptions, one of which is applicable here. Furthermore, (A) is not a correct statement of law. Jeopardy attaches in a jury trial when the jury is impaneled and sworn in, even if it has not yet heard any evidence. (B) is incorrect because the judge's finding at the preliminary hearing stage appears to be an honest error rather than bad faith conduct. In the absence of bad faith conduct by the judge or prosecutor designed to force the defendant to seek a mistrial, the defendant's securing of a mistrial does not preclude a retrial. (D) is incorrect because the right to be free of double jeopardy creates a bar as soon as the defendant is retried for the same offense, rather than on her conviction.

In order to be guilty of common law burglary, one must break and enter a dwelling with: A Recklessness about the consequences that could occur if someone was inside the home B No particular mental state; the act of breaking and entering is sufficient to establish burglary C The specific intent to commit a felony therein D Knowledge that one lacks authority to enter the dwelling

C. Burglary is a specific intent crime. A specific intent crime requires the doing of the criminal act with a specific intent or objective. The common law definition of burglary is the breaking and entering of the dwelling of another at nighttime with the intent of committing a felony therein. Because burglary requires the intent to commit a felony at the time of the entering, it is a specific intent crime.

Which of the following is true regarding the provocation required to reduce murder to manslaughter? A As long as the defendant was in fact provoked, it does not matter if an ordinary person would have been provoked under the circumstances. B Whether the defendant had sufficient time to cool off is measured solely by an "ordinary person," objective standard. C The provocation must be such as to raise a sudden and intense passion in the mind of an ordinary person, and the defendant must have in fact been provoked. D As long as an ordinary person would have been provoked, it does not matter whether the defendant was in fact provoked.

C. For reducing murder to manslaughter, the provocation must be such as to raise a sudden and intense passion in the mind of an ordinary person, and the defendant must have in fact been provoked. The provocation requirement for voluntary manslaughter has both objective and subjective elements. The provocation must be such that an ordinary person would have been provoked, and the defendant must have been so provoked. There are also objective and subjective elements as to the "cooling off" issue. If sufficient time elapsed between the provocation and the killing such that a reasonable person would have cooled off, or if the defendant in fact "cooled off," a reduction to voluntary manslaughter is not available.

Under the plain view exception to the warrant requirement, which of the following is NOT required? A The police must be legitimately on the premises where the item is found. B The item must be evidence, contraband, or a fruit or instrumentality of a crime. C The police must have inadvertently discovered the item. D It must be immediately apparent that the item is evidence, contraband, or a fruit or instrumentality of a crime.

C. For the warrantless seizure of an item under the plain view exception, it is not required that the item be inadvertently discovered. Formerly, this exception to the warrant requirement applied only if the item was inadvertently discovered. Inadvertence, however, is no longer a requirement. To make a warrantless seizure, the police (i) must be legitimately on the premises where the item is found; (ii) the item must be evidence, contraband, or a fruit or instrumentality of a crime; (iii) the item must be in plain view; and (iv) it must be immediately apparent (i.e., probable cause) that the item is evidence, contraband, or a fruit or instrumentality of a crime.

If a detainee requests the presence of counsel at an interrogation under Miranda: A The prohibition against questioning the detainee ends immediately after he is released. B The request must be clear to the police officer present. C The police may continue to interrogate if the request is ambiguous. D The request can be met by halting the interrogation to allow the detainee to consult with counsel and reinitiating the interrogation once counsel leaves.

C. If a detainee requests the presence of counsel at interrogation, under Miranda the police may continue to interrogate if the request is ambiguous. Once a detainee has expressed an unequivocal desire for counsel, all questioning must cease. However, when the request is ambiguous, the police may continue to interrogate until the detainee gives an unambiguous request. The police may also ask clarifying questions when the request is ambiguous. It is inaccurate to state that the detainee's request for the presence of counsel must be clear to the police officer present. The test is objective—the request must be sufficiently clear that a reasonable police officer in the same situation would understand the statement to be a request for counsel. Following a request for the presence of counsel by a detainee, the prohibition against questioning the detainee does NOT end immediately after he is released. Instead, the prohibition lasts the entire time the detainee is in custody for interrogation, plus 14 more days after the detainee returns to his normal life. After that point, the detainee may be questioned about the same matter upon receiving a fresh set of Miranda warnings. Finally, if there has been a request for counsel, it is not enough for the police merely to halt the interrogation to allow the detainee to consult with counsel and then reinitiate the interrogation once counsel leaves. Mere consultation with counsel prior to questioning does not satisfy the right to counsel; the police cannot resume questioning the detainee in the absence of counsel.

A man and a woman were arrested and charged with conspiring to blow up a federal government building. After being given Miranda warnings, they were questioned separately and each of them gave a written confession. The confessions interlocked with each other, implicating both of the defendants as being involved in every stage of the conspiracy. Subsequently, the woman attempted to retract her confession, claiming that it was false. At a preliminary hearing, the judge rejected her claim. Both defendants were tried together, and the prosecutor introduced both confessions into evidence. At trial, the woman testified that she was not involved in any conspiracy and that her confession was fabricated. Both defendants were found guilty by the jury.The woman challenged her conviction on appeal because of the admission of the man's confession. If the woman succeeds, what is the likely reason? A The man's confession was more incriminatory to her than her own confession. B The jury was not instructed to consider the man's confession as evidence only of his guilt and not of the woman's. C The man refused to testify at trial and therefore was not subject to cross-examination regarding his confession. D The man testified at trial and was subject to cross-examination but denied making the confession attributed to him.

C. If the woman prevails in her challenge to the admission of the man's confession it will be because the man could not be cross-examined regarding his confession. Under the Sixth Amendment, a defendant in a criminal prosecution has the right to confront adverse witnesses at trial. If two persons are tried together and one has given a confession that implicates the other, the right of confrontation generally prohibits the use of that statement because the other defendant cannot compel the confessing co-defendant to take the stand for cross-examination. A co-defendant's confession is inadmissible even when it interlocks with the defendant's own confession, which is admitted. If the man refused to take the stand and subject himself to cross-examination, his confession was not properly admitted because it violated the woman's Confrontation Clause rights. (A) is incorrect because the fact that the man's confession incriminates the woman more than her own confession is not relevant. Just the interlocking nature of the man's confession with the woman's confession makes it more damaging by making it harder for the woman to claim that her confession was false. (B) is incorrect because the Supreme Court has held that instructing the jury to consider the confession only as going to the guilt of the confessing defendant is inadequate to avoid Confrontation Clause problems, because the risk that the jury will not follow the limiting instructions is too great in this context. (D) is incorrect. Confessions of a co-defendant may be admitted if (i) all portions referring to the other defendant can be eliminated (so that there is no indication of that defendant's involvement), (ii) the confessing defendant takes the stand and subjects himself to cross-examination regarding the truth or falsity of the statement, or (iii) the confession of the nontestifying co-defendant is being used to rebut the defendant's claim that his confession was obtained coercively, and the jury is instructed as to that purpose. Even if the co-defendant denies ever having made the confession, as stated in choice (D), the opportunity at trial to cross-examine the co-defendant satisfies the Confrontation Clause.

A defendant was convicted after a jury trial of violation of federal statutes prohibiting the sale of automatic weapons to foreign nationals. It was established at trial that the defendant had purchased a number of stolen United States Army heavy machine guns and attempted to ship them abroad. The trial court expressly based its imposition of the maximum possible sentence for the conviction on the defendant's refusal to reveal the names of the persons from whom he purchased the stolen weapons. His counsel argues that this consideration is reversible error. If the defendant appeals the sentence imposed, what should the appeals court do? A Reverse the trial court, because the consideration of the defendant's silence violates his Fifth Amendment privilege against self-incrimination. B Reverse the trial court, because the consideration of collateral circumstances in sentencing violates his due process rights. C Affirm the trial court, because the right to remain silent granted by the Fifth Amendment does not include the right to protect others from incrimination. D Affirm the trial court, because citizens must report violations of the criminal statutes.

C. The appeals court should affirm the trial court because the right to remain silent does not include the right to protect others from incrimination. The defendant was not privileged to refuse revealing the names of the stolen weapon sellers. The United States Supreme Court held, in Roberts v. United States (1980), that a defendant's refusal to cooperate with an investigation of the criminal conspiracy of which he was a member may properly be considered in imposing sentence. This is because the Fifth Amendment right to remain silent does not afford a privilege to refuse to incriminate others. (C) is therefore correct and (A) is incorrect. (B) is incorrect because the court's consideration of the defendant's refusal to cooperate does not violate due process. (D) is not an accurate statement of the law.

The constitutional prohibition against double jeopardy prohibits: A an appeal following the grant of a motion to set aside the jury's verdict B an appeal of a case dismissed on grounds other than merits-based grounds C a person from being tried for the same conduct by both a state and one of its municipalities D a person from being tried for the same conduct by both a state and the federal government

C. The constitutional prohibition against double jeopardy prohibits a person from being tried for the same conduct by both a state and one of its municipalities. A state and one of its municipalities are considered the same sovereign, and double jeopardy applies to trials of the same conduct by the same sovereign. On the other hand, double jeopardy does not apply to trials by separate sovereigns, so a person MAY be tried for the same conduct by both a state and the federal government. Even after jeopardy has attached, the Double Jeopardy Clause does not prohibit an appeal of a case dismissed on grounds other than merits-based grounds. The prosecution may appeal any dismissal based on the defendant's motion that does not constitute an acquittal on the merits. Similarly, the Double Jeopardy Clause does not bar an appeal following the grant of the defendant's motion to set aside the jury's verdict. An appeal by the prosecution is not barred if a successful appeal would not require a retrial, and if an appellate court overrules the grant of a motion to set aside a jury verdict, no new trial is held; rather, the original jury's verdict is reinstated.

In criminal cases, certain rights may be waived. Which of the following statements is correct regarding waiver of the right to counsel under the Fifth and Sixth Amendments? A Neither the Fifth nor Sixth Amendment right to counsel can be waived, because the rights provided by each are fundamental to a fair trial. B The Fifth Amendment right to counsel may be waived by a knowing and voluntary waiver, but the Sixth Amendment right cannot be waived. C The Fifth and Sixth Amendment right to counsel may each be waived by a knowing and voluntary waiver. D The Sixth Amendment right to counsel may be waived by a knowing and voluntary waiver, but the Fifth Amendment right cannot be waived.

C. The correct choice is the Fifth and Sixth Amendment right to counsel may each be waived by a knowing and voluntary waiver. Whether a waiver is knowing and voluntary is judged by a totality of the circumstances. Note that as to the Fifth Amendment, if a defendant is given Miranda warnings (e.g., notice of a right to counsel and to remain silent) and chooses to speak, the court usually will find a valid waiver. Indeed, to exercise the Fifth Amendment right to counsel, the defendant must make an unambiguous and specific request for counsel. Words like "Maybe I should talk to a lawyer," do not equal a request to do so. Note also that the Supreme Court has held that waiver of the Sixth Amendment right to counsel does not require the presence of counsel, unless the defendant had already requested counsel.

A woman was arrested, given Miranda warnings, and questioned about an armed robbery. After she asked to speak with an attorney, the police stopped questioning her about the robbery. Several hours later, the police gave the woman a fresh set of Miranda warnings and began to question her about a different robbery. She did not repeat her request for an attorney and instead made several incriminating statements about the robbery. At the woman's trial for the robbery for which she made incriminating statements, the prosecution seeks to have her statements introduced into evidence. If the woman's attorney objects on appropriate grounds, how should the court rule? A Overrule the objection, because the police did not badger the woman into confessing. B Overrule the objection, because the woman did not renew her request for an attorney after receiving fresh Miranda warnings. C Sustain the objection, because the police did not honor the woman's request. D Sustain the objection, because a confession obtained in violation of a defendant's Miranda rights but otherwise voluntary may be used against the defendant.

C. The court should sustain the objection because the police did not honor the woman's request for an attorney. At any time prior to or during a custodial interrogation, the accused may invoke a Miranda (Fifth Amendment) right to counsel. If the accused invokes this right, all questioning must cease until the accused is provided with an attorney or initiates further questioning himself. Thus, the police questioning of the woman about the robbery was improper, and she can have her statements excluded. (A) is incorrect. After receiving Miranda warnings, if an accused invokes the right to remain silent, the police cannot badger the accused. However, courts have ruled that if the police scrupulously honor the request, they can rewarn the accused and later resume questioning, at least about a different crime. Here, however, the accused did not simply invoke the right to remain silent, but rather requested an attorney. After such a request, as indicated above, all questioning must cease. (B) is incorrect because the accused does not need to reassert the right to an attorney; all questioning must stop until the accused is provided an attorney or resumes the questioning herself. (D) is incorrect. It is stating the rule for impeachment - a confession obtained in violation of a defendant's Miranda rights but otherwise voluntary may be used against the defendant for purposes of impeachment, but there is no such rule for use of the confession for other purposes.

A driver was operating her car on a city street when she was stopped by a police officer for speeding. As the police officer reached the driver's car, he saw her put something into her purse. The officer told the driver, "Ma'am, you were speeding; that's why I stopped you. I'd like your driver's license, and, by the way, what did you just put into your purse?" The driver responded, "It's just a marijuana cigarette, but don't worry, I've only had two and my driving judgment hasn't been impaired." The officer took her purse, removed the "joint," and charged the driver with possession of marijuana as well as speeding. At the driver's trial for marijuana possession, the prosecution seeks to introduce the marijuana cigarette into evidence. The driver's attorney moves to suppress the evidence. The defense motion should be: A Granted, because the cigarette is fruit of the poisonous tree. B Granted, because the police officer did not have a valid search warrant. C Denied, because the police officer's asking about the contents of the driver's purse did not constitute custodial interrogation. D Denied, provided the police officer had a reasonable suspicion of criminal activity.

C. The defense motion should be denied because the driver was not in custody when she made the statement. Persons temporarily detained for routine traffic stops are not in custody for Miranda purposes. Therefore, the driver was not entitled to Miranda warnings, and her statement about the marijuana was not tainted. Her statement thus properly provided the probable cause for the search of her purse. (A) is therefore wrong. (B) is wrong because this case falls within the automobile exception to the warrant requirement. Under that exception, if the police have probable cause to believe that a vehicle contains contraband or fruits, instrumentalities, or evidence of a crime, they may search the vehicle, including the driver's belongings, without a warrant. Here, the driver's response established probable cause to search her purse.(D) states the test for a stop, not a search. An automobile search requires probable cause.

Acting with probable cause, the police arrested a man in connection with the armed robbery of a liquor store. After being given Miranda warnings, the man confessed to the robbery but denied his involvement with several other recent armed robberies of businesses in the area. He was formally charged with the one robbery and put into a cell with a paid informant working undercover for the police. The informant had been instructed to find out what he could about the other robberies but not to ask any questions. The informant began talking about a convenience store robbery in which a bystander was shot and seriously injured by the robber, and he deliberately misstated how it happened. The man, unaware that his cell mate was an informant, interrupted to correct him, bragging that he knew what really happened because he was there, and proceeded to make incriminating statements about the robbery. The man was subsequently charged with armed robbery and attempted murder in the convenience store robbery. At a motion-to-suppress hearing on that charge, if the man's attorney moves to exclude the statements made to the informant, should the motion be granted? A Yes, because the informant deliberately elicited incriminating statements in violation of the man's Sixth Amendment right to counsel. B Yes, because the informant's conduct constituted custodial interrogation in violation of the man's Fifth Amendment privilege against self-incrimination. C No, because the man had not yet been charged with the robbery of the convenience store when he made the statements to the informant. D No, because the informant's conduct did not constitute interrogation.

C. The man's motion should be denied because neither his Fifth nor Sixth Amendment rights were violated by the informant's conduct. The Sixth Amendment right to counsel applies to all critical stages of a criminal prosecution after formal proceedings have begun, but does not apply in precharge custodial interrogations. Because this right is "offense specific," the fact that the right to counsel has attached for one charge does not bar questioning without counsel for an unrelated charge. Because the man has not been charged with the convenience store robbery, his Sixth Amendment right to counsel has not been violated. The Fifth Amendment privilege against self-incrimination requires Miranda warnings and a valid waiver before any statement made by the accused during custodial interrogation can be admitted. However, this requirement does not apply where interrogation is by an informant who the defendant does not know is working for the police, because the coercive atmosphere of police-dominated interrogation is not present. [Illinois v. Perkins (1990)] Because the man was not aware of the informant's status, the informant's conduct did not constitute a police interrogation. (A) is wrong despite the fact that the informant's conduct may have been deliberately designed to elicit incriminating remarks. As discussed above, the man's right to counsel did not attach for purposes of the convenience store robbery. (B) is incorrect because, as discussed above, the Miranda warnings need not be given before questioning by a cellmate working covertly for the police. (D) is incorrect because interrogation refers not only to express questioning, but also to any words or actions on the part of the police that the police should know are reasonably likely to elicit an incriminating response from the suspect. Here, the informant, working for the police, made statements about the convenience store robbery that were intended to, and reasonably likely to, prompt a response from his cellmate. Hence, it is not the absence of "interrogation" that avoids the Miranda problem, but the fact that the man did not know that his cellmate was working for the police.

The police received information linking a man to drug trafficking and went to the man's residence, where he lived with his mother. The police found the mother at home, and she told them that her son was not expected back until later. The police informed the mother that they suspected the man of selling drugs and asked if they could search his room. She replied, "I'm finished with that no-good bum; not only is he into drugs, but he has been stealing my money to pay for them, and all the time I'm making his bed and fixing his food. You can search his room. He likes to keep his private stuff under his pillow. I hope he goes to jail." The police searched the man's room and discovered a quantity of marijuana under the pillow of his bed. If before trial the man's attorney moves to suppress the marijuana on grounds that the search was invalid, should the court grant the motion? response - incorrect A Yes, because the man had a legitimate expectation of privacy in the area searched, and the police did not have a warrant. B Yes, because the man's mother's consent was given at a time when police knew her interests were in conflict with the man's. C No, because the man's mother had the authority to consent to the search of his room. D No, because with the mother's statement the police had probable cause to search the room.

C. The man's motion to suppress should be denied because his mother had authority to consent to the search of his room. A search of a residence can be based on the voluntary consent of the occupant. Where a parent has general access to a room occupied by a son or daughter, the parent can give a valid consent to a general search of the room even if the son or daughter is an adult. The facts in the question indicate that the man's mother had general access to his room ("and all the time I'm making his bed"). Therefore, her consent is valid and eliminates the need for probable cause and a warrant. (A) is wrong. The man had a legitimate expectation of privacy in the area searched, but the consent of his mother eliminated the need for a warrant. (B) is wrong. At one time, some courts required an "amicable relationship" between the parties before the police could rely on a third party's consent. The "amicable relationship requirement" is no longer recognized by the courts. (D) is not a good answer. It is true that with the mother's statement the police had probable cause to search the man's room. However, probable cause alone would not validate the search. The police would need probable cause plus a warrant or a valid consent. In this question the search would have to be based on consent.

A college student was the sole lifetime beneficiary under a large trust administered by a banker. The student received a large monthly distribution from the trust, and whenever he ran short, he simply called the banker for extra funds, because the trust provided that the student was to receive whatever he needed from income or principal. The student's roommate found out about the trust arrangement and decided to see if he could make it pay off for him. The roommate sent an e-mail to the banker, which appeared to be from the student, and which asked for several thousand dollars to cover medical expenses. The e-mail further stated that, since he was in the hospital, the student would send his roommate to pick up the cash. The next day, the roommate showed up at the banker's office and obtained the money on the promise that he would take it to the student. The roommate absconded with the funds. When the roommate obtained the cash from the banker, what crime did the roommate commit? A False pretenses. B Embezzlement. C Larceny by trick. D Larceny.

C. The roommate committed larceny by trick because the banker's consent to the roommate's taking the money was induced by the misrepresentation that the roommate would take the money to the student/beneficiary. Larceny consists of a taking and carrying away of tangible personal property of another by trespass, with intent to permanently (or for an unreasonable time) deprive the person of his interest in the property. If the person in possession of property has not consented to the taking of it by the defendant, the taking is trespassory. However, if the victim consents to the defendant's taking possession of the property, but such consent has been induced by a misrepresentation, the consent is not valid. Under such circumstances, the larceny is called larceny by trick. Here, the roommate obtained the money from the banker on the promise that he would take it to the student/beneficiary. This misrepresentation induced the banker to give possession of the money to the roommate. The roommate then proceeded to take the money and carry it away, intending all the while to permanently deprive one who had a possessory interest superior to the roommate's of his interest in the money. Thus, all the elements of larceny are present. Because the original wrongful taking resulted from consent induced by misrepresentation, the specific larceny committed by the roommate is more precisely characterized as larceny by trick. Consequently, although the roommate has in fact committed larceny, (C) is a better answer than (D). Regarding (A), false pretenses consists of obtaining title to the property of another by an intentional (or knowing) false statement of past or existing fact, with intent to defraud the other. If a victim intends to convey only possession of the property to the defendant, the offense is larceny by trick. However, if the victim intends to convey title, the offense is false pretenses. Here, the banker intended to convey possession of the money to the roommate so that he could give the money to the student/beneficiary. The banker did not intend to convey title to the roommate. Because the roommate did not obtain title by means of his misrepresentation but simply obtained possession, the offense of false pretenses was not committed. (B) is incorrect because embezzlement is the fraudulent conversion of property of another by a person in lawful possession of that property. In embezzlement, the misappropriation of the property occurs while the defendant has lawful possession of it. In larceny, the misappropriation occurs generally at the time the defendant obtains wrongful possession of the property. The roommate did not have lawful possession of the money because his possession of the money resulted from his misrepresentation to the banker. Thus, the roommate's taking of the money was wrongful from the outset. Because the roommate had wrongful, rather than lawful, possession of the money, there was no embezzlement.

Traditionally, which of the following is a defense to solicitation? A The solicitation could not have been successful due to actual circumstances. B There was no overt act committed in furtherance of the solicitation. C The solicitor is exempt from liability for the completed crime. D The solicitor withdrew the solicitation.

C. The solicitor would have a defense if she could not be guilty of the intended crime because of a legislative intent to exempt her. It is not a defense that the solicitation could not have been successful due to actual circumstances surrounding the crime. (This is factual impossibility.) The culpability of the solicitor is measured by the circumstances as she believed them to be. It is generally not a defense that the solicitation has been withdrawn once it has been made. However, note that the M.P.C. recognizes renunciation as a defense if the defendant prevents the commission of the crime, such as by persuading the person solicited not to commit the crime. For solicitation, no overt act in furtherance of the solicitation is required. (An overt act is usually required for conspiracy.)

If the police do not have probable cause to make an arrest, which of the following statements best reflects their ability to stop a person for investigatory purposes? A The police may stop a person for investigatory purposes as long as they have at least a scintilla of suspicion. B The police may not stop a person for investigatory purposes absent probable cause to arrest. C The police may stop a person for investigatory purposes if they have at least reasonable suspicion to investigate based on articulable facts. D The police may not stop a person for investigatory purposes absent probable cause to investigate.

C. The statement that the police may stop a person for investigatory purposes if they have at least reasonable suspicion to investigate based on articulable facts reflects the correct standard for making an investigatory stop, also known as a Terry stop. Such stops should be brief and for no longer than necessary to verify or refute the officer's suspcion. Reasonable suspicion requires less than probable cause, but more than a vague notion or gut feeling. The suspicion can be based on personal observation, reliable tips, police flyers, a bulletin, or the like. Whether the standard is met is judged under the totality of the circumstances. The choices indicating that the police may not stop a person for investigatory purposes absent probable cause are both incorrect, because, as indicated above, probable cause is not required. Reasonable suspicion is sufficient. Moreover, the choice requiring probable cause to arrest also is incorrect because a stop can be made for suspicion of criminal activity; investigatory stops are not limited to cases where the police already believe an arrest is warranted. It is not correct that the police may stop a person for investigatory purposes as long as they have at least a scintilla of suspicion, because a scintilla is a very small amount. The scintilla of proof standard is too little suspicion to satisfy the Fourth Amendment requirement that searches and seizures be reasonable. It is sometimes used as the standard for determining whether a case may be taken from a jury and decided on a motion for summary judgment (i.e., if there is a scintilla of evidence regarding a material issue, the case should not be taken from the jury).

A student and a few of his friends were making their way to spring break. Along the way, the old van that they were driving broke down. Not wanting to miss any part of spring break festivities, the student asked the mechanic on duty at the repair shop for a rush job. The mechanic provided the student with a repair estimate, and the student, on the basis of the estimate, authorized the repair and promised to pay when he came back to pick up the van. When the mechanic called the student to tell him that the van was repaired, the student, rather than paying for the repair, told one of his friends that the mechanic had agreed to finance the repair charges and that the only thing left to do was pick up the van in the garage's parking lot. The student handed the friend a key to the van and told him to go pick the van up so that they could continue their trip to spring break. The friend did so. The mechanic makes a criminal complaint against the student for larceny of the van. If the case is prosecuted, will the student likely be found guilty? response - incorrect A No, because it was the student's van to begin with. B No, because the friend took the van. C Yes, because the friend took the van from the mechanic without the mechanic's knowledge or permission. D Yes, because the student promised to pay the mechanic for his work when he came to get the van.

C. The student will most likely be found guilty. Larceny is the taking and carrying away of the personal property of "another" with the intent to permanently deprive the other person of the property. It is possible to commit larceny of your own property if another person, such as a bailee, has a superior right to possession of the property at that time. Because the mechanic had a right to possession of the van until he was paid, the student committed larceny when he had his friend take the van without the mechanic's consent. (B) is wrong because a person can be guilty even though he did not personally engage in the behavior if he acts through an innocent agent. (D) is also incorrect. The student is guilty, but not for the reason stated in (D). He would be guilty even if he had not made the promise to pay for the van; he incurred an obligation to pay by having the repairs done.

Which of the following is available as a defense to specific intent crimes but not general intent crimes? A Mistake of fact B Insanity C Voluntary intoxication D Involuntary intoxication

C. Voluntary intoxication is available as a defense to a specific intent crime only. The defense may be used to show that a defendant was unable to form the requisite intent due to intoxication. Involuntary intoxication and insanity are available as defenses to both specific intent and general intent crimes. Mistake of fact is available as a defense to both specific intent and general intent crimes. However, note that a mistake of fact must be reasonable to be a defense to a general intent crime. An unreasonable mistake of fact is a defense to a specific intent crime only.

A police officer was given a tip about a blonde male living in a nearby trailer park who was selling narcotics. The officer immediately drove to the trailer park and obtained from the manager the names of six blonde males who had trailers or mobile homes in the trailer park. At the first lot, the officer knocked on the defendant's door, announced that he was a police officer, and asked to talk to the defendant. The defendant's girlfriend, who did not live there but had been visiting, told the officer that the defendant would not be back for some time. The officer, believing that the girlfriend lived there, told her that he suspected that the defendant was dealing drugs and asked her if he could look around a little. The girlfriend said, "Sure, why not?" and let the officer in. After seeing nothing in the main living area, he went into the small back bedroom and opened several small storage compartments. In the corner of one of the compartments, he found an opaque bag. On opening it, he observed that it contained what appeared to be marijuana and confiscated the bag. Shortly thereafter, the defendant was arrested and charged with possession of narcotics with intent to distribute, a felony. On a motion by the defendant's attorney to suppress the introduction of the marijuana into evidence, how is the court likely to rule? A For the defendant, because his girlfriend did not live in the trailer. B For the defendant, because the search exceeded the scope of the consent. C Against the defendant, because mobile homes fall within the automobile exception to the warrant requirement. D Against the defendant, because the officer reasonably believed that the defendant's girlfriend lived in the trailer.

D The court should deny the defendant's motion because the officer reasonably believed that the defendant's girlfriend lived in the trailer, making the search valid. Under the exclusionary rule, evidence obtained from an unconstitutional search must be excluded from trial. To be valid, searches must be reasonable. The Supreme Court has held that most searches are unreasonable unless the police obtain a warrant before searching. However, there are six categories of searches that the Court has held to be reasonable without a warrant. One such category is searches conducted pursuant to consent. To fall within this exception to the warrant requirement, consent must be given by one who appears to have an apparent right to use or occupy the premises and the search cannot go beyond the scope of the consent given. The consent is valid as long as the police reasonably believed that the person who gave the consent had the authority to do so, and the scope of the consent is limited only to areas to which a reasonable person under the circumstances would believe it extends. Here, the girlfriend's consent was valid because the officer believed that she lived there. His belief appears to be reasonable because she answered the door, knew of the defendant's whereabouts, and readily consented to the search. Therefore, the search was valid under the consent exception and the evidence should not be excluded. (A) is incorrect because consent is not invalid merely because the person who gave it did not actually have authority to do so; the police need only reasonably believe that the person had authority to consent, and as explained above, it was reasonable for the officer to believe that the defendant's girlfriend had authority here. (B) is incorrect because the scope of consent extends to any area where a reasonable person under the circumstances would assume it extends. Because the officer told the girlfriend that he suspected the defendant of dealing drugs, it was reasonable to assume that he was looking for drugs and so would probably look in even small containers. (C) is incorrect because it appears that the defendant's trailer would not fall within the automobile exception. Certain searches of automobiles are excluded from the requirement of a warrant because the Supreme Court has held that people have a lesser expectation of privacy in an automobile than in other areas and automobiles are likely to disappear before a warrant can be acquired. The automobile exception extends not only to cars, but also to other vehicles that are readily mobile and as to which there is a lesser expectation of privacy. However, nothing in the facts here indicates that the defendant's mobile home may readily be moved, and because it appears to be the defendant's regular home rather than a vehicle, it is doubtful that the Court would find the requisite lesser expectation of privacy. Therefore, the trailer would not fall within the automobile exception to the warrant requirement.

Irresistible Impulse test

D gets acquittal if b/c of mental illness, D was unable to control his actions or conform his conduct to the law

Durham Test (NH only)

D gets acquittal if crime was the product of his mental illness (crime would not have been committed but for his mental illness)

ALI or MPC Test

D gets acquittal if he had a mental disease or defect, and, as a result, he lacked the substantial capacity to either (i) appreciate the criminality of his conduct; or (ii) conform his conduct to the requirements of the law [combination of M'Naghten and irresistible impulse]

M'Naghten Rule

D had (i) disease of mind; (ii) caused a defect of reason; (iii) such that D lacked ability at time of his actions to know the wrongfulness of his actions or understand the nature and quality of his actions [delusions, belief that his actions are morally right, loss of control]

What separates voluntary manslaughter from murder is: A The killing is "caused" by the commission of a felony B The killing is "caused" by the commission of a crime not amounting to a felony C The lack of express malice D Adequate provocation

D. A killing committed under adequate provocation is voluntary manslaughter. A killing committed during the commission of an unlawful act not amounting to a felony is involuntary manslaughter. Committing a felony that results in death is murder. The intent to commit a felony satisfies the malice requirement to classify a killing as murder. A killing committed without express malice may be murder if malice is implied. "Malice aforethought" for common law murder can be satisfied by (i) the intent to kill; (ii) the intent to inflict great bodily injury; (iii) a reckless indifference to an unjustifiably high risk to human life; or (iv) the intent to commit a felony. The intent to kill is deemed to be "express malice," whereas, in the latter three, malice is implied. Although "implied," the latter three states of mind also satisfy the malice requirement for common law murder.

In a majority of jurisdictions, an assault is defined either as: A an attempt to commit an offensive touching, or the commission of an offensive touching that does not cause bodily injury B an offensive touching that is caused negligently rather than intentionally, or the intentional creation, other than by mere words, of a reasonable apprehension in the mind of the victim of imminent bodily harm C an offensive touching that does not cause any bodily injury, or the intentional creation, other than by mere words, of a reasonable apprehension in the mind of a victim of imminent bodily harm D an attempt to commit an offensive touching, or the intentional creation, other than by mere words, of a reasonable apprehension in the mind of the victim of imminent bodily harm

D. In a majority of jurisdictions, an assault is either an attempt to commit an offensive touching, or the intentional creation, other than by mere words, of a reasonable apprehension in the mind of the victim of imminent bodily harm. Generally, battery is defined as an unlawful application of force to the person of another resulting in either bodily injury or, at a minimum, an offensive touching. Therefore, the first definition of assault can also be explained as an attempt to commit a battery. If any offensive touching occurs, the crime is battery, regardless of intent, and not assault. The crime of battery does not require bodily injury; any offensive touching suffices. The crime of battery is not downgraded to an assault because no injury occurs.

Which of the following statements is correct regarding the ability of police officers to stop automobiles for investigatory purposes? A The police may stop an automobile for investigatory purposes without cause because automobiles are not areas protected by the Fourth Amendment. B The police may not stop an automobile for investigatory purposes unless they have probable cause to believe that the driver has broken a law. C The police may not stop an automobile for investigatory purposes unless they have reasonable suspicion that the driver has broken a law. D The police may stop automobiles for investigatory purposes even without reasonable suspicion if they make the stops on a neutral, articulable basis to investigate a problem closely related to the mobility of automobiles.

D. It is true that the police may stop automobiles on a neutral, articulable basis to investigate a problem closely related to the mobility of automobiles. Generally, to make an investigatory stop of an automobile, the police must have at least reasonable suspicion that the driver or an occupant violated or was about to violate some law (i.e., the normal Terry standard). However, the Supreme Court has recognized that the mobility of automobiles can cause special problems, and allow the police to set up roadblocks to stop automobiles even without individualized suspicion to investigate these problems. A good example would be a roadblock that stops every car to determine whether the drivers are intoxicated. The police do not need to have probable cause to believe that the driver has broken the law to stop an automobile for investigatory purposes, because automobiles can be stopped on reasonable suspicion or even without any individualized suspicion, as discussed above. The police do not need to have even reasonable suspicion that the driver has broken the law to stop an automobile for investigatory purposes. First, the driver is not the only possible object of reasonable suspicion. And second, as discussed above, automobiles can be stopped even without any individualized suspicion, to investigate a problem closely related to automobiles and their mobility, as long as the stops are made on a neutral and articulable basis. The choice indicating that automobiles are not areas protected by the Fourth Amendment is incorrect. The Fourth Amendment protects people rather than places. And the courts have held that people do have a privacy interest (albeit a diminished one) when in their automobiles and are protected by the Fourth Amendment.

Which of the following will not establish the general intent requirement of a crime? A The doing of the act B The intent to cause a harmful result to a different person or object C Proof that the defendant is acting in a proscribed way and that any attendant circumstances required by the crime are present D Motive

D. Motive will not establish the general intent requirement of a crime. Motive is merely a reason or explanation for committing the offense, and is generally immaterial to substantive criminal law. A jury can infer the required general intent merely from the doing of the act. It is not necessary that evidence specifically proving the general intent be offered by the prosecution. The intent to cause a harmful result to a different person or object is sufficient to establish intent to cause a similar harmful result to the person or object actually harmed. This is known as transferred intent. General intent requires that the defendant be aware that he is acting in a proscribed way and that any attendant circumstances required by the crime are present. Thus, proof that this is true will establish general intent.

A husband who believed that his wife was having an affair with his brother hired an arsonist to burn down the brother's house. They planned for the husband to take his brother to a ballgame so that the arsonist would be able to set the house on fire without detection. After the husband and brother left for the ballgame, however, the arsonist decided to abandon the plan and immediately left town without doing anything further. When the husband returned from the ballgame with the brother, he saw the house still standing and blurted out what was supposed to have happened. The husband and the arsonist were arrested and charged with conspiracy to commit arson. At the arsonist's trial, his attorney argued that he was innocent of the conspiracy because he decided not to go ahead with the plan, and nothing criminal had in fact occurred. At common law, how should a jury find the arsonist? response - incorrect A Not guilty of conspiracy, because going to a ballgame is not a criminal overt act. B Not guilty of conspiracy, because the husband, not the arsonist, committed the overt act. C Guilty, because the husband executed his part of the plan. D Guilty, because the arsonist agreed to set the brother's house on fire.

D. The arsonist should be found guilty. A conspiracy is a combination or agreement between two or more persons to accomplish some criminal or unlawful purpose, or to accomplish a lawful act by unlawful means. The mens rea required for conspiracy is specific intent, in that both parties must intend to agree to accomplish some criminal or unlawful purpose. Thus, once the arsonist was hired by the husband and they came up with a plan to burn down the brother's house, the crime of conspiracy was completed. (C) is incorrect because it implies that carrying out the plan by at least one party is required; the conspiracy was complete even before the husband fulfilled his duties under the plan. Note that, while most states now require an overt act for conspiracy, the common law version does not. (A) is incorrect. Even if an overt act were required, it need not be in and of itself criminal. (B) is also incorrect. If an overt act were required, it need only be performed by one of the co-conspirators, not necessarily the conspirator on trial.

Which of the following presents the strongest intervening event to relieve the defendant of homicide liability? A The defendant stabs the victim; the victim commits suicide due to the resulting pain. B The defendant stabs the victim, who is treated at a hospital. Due to negligent care, the victim contracts an infection and dies. C The defendant stabs the victim; the victim refuses medical treatment and dies. D The defendant stabs the victim, who is treated for two hours at a hospital. On his way home from the hospital, the victim is killed in an automobile accident.

D. The case where the defendant stabs the victim, who after being treated at the hospital, is killed in an automobile accident, is likely to present an intervening event sufficient to relieve the defendant of homicide liability. As a general rule, an intervening act will shield the defendant from liability if the act is a mere coincidence or is outside the foreseeable sphere of risk created by the defendant's act. The automobile accident presents a case of a mere coincidence that is outside the foreseeable sphere of risk. It could be said that, but for the defendant's act of stabbing the victim, the victim probably would not have been in an automobile accident. However, the connection between the stabbing and the automobile accident is too tenuous and unforeseeable to allow for homicide liability against the defendant. Acts by third parties that are within the foreseeable sphere of risk created by the defendant's act will not be sufficient intervening acts to relieve the defendant of homicide. Unforeseeable risks, however, will be sufficient. Negligent medical care (as opposed to grossly negligent or intentional mistreatment) is not a sufficient intervening act to relieve the defendant of homicide liability. Acts by the victim, such as refusing medical care and committing suicide, generally will not be considered intervening acts.

Acting on a hunch, a police officer went to a young woman's apartment, broke in, and searched it. The officer found exactly what she was looking for under the woman's bed: a sack filled with jewels. The attached note read, "Sweetheart, here are the goods from the estate heist. Your loving boyfriend." It was well known in the community that the woman's boyfriend was a jewel thief. The officer also knew that the estate of a local socialite had been burglarized three days ago. Just as the officer finished reading the note, the woman returned. The officer immediately placed the woman under arrest as an accessory to the estate burglary. Based on the evidence obtained from the woman's apartment, a search warrant was issued for her boyfriend's apartment. The search yielded burglar tools and more jewels from the estate. The boyfriend was immediately arrested and charged with the estate burglary. At the boyfriend's trial for the estate burglary, his attorney files a motion to suppress the evidence consisting of the bag of jewels and note, the tools, and the jewels from the boyfriend's apartment. How should the court rule on the motion? response - incorrect A Grant the motion as to the bag of jewels and note, but deny it as to the evidence found in the boyfriend's apartment. B Grant the motion, because all of this evidence is fruit of the poisonous tree. C Deny the motion, because the police would have caught the boyfriend with the goods eventually. D Deny the motion, because the police had a warrant to search the boyfriend's apartment.

D. The court should deny the motion to suppress because the police had a warrant to search the boyfriend's home. The boyfriend's expectation of privacy extended only to his own home, which was searched under a warrant. He does not have standing to assert a Fourth Amendment claim regarding the search of his girlfriend's apartment because her apartment was not his home, and he did not own it or have a right to possession of it. Thus, (A) is incorrect. Because the boyfriend cannot object to the search that provided the probable cause for the search of his apartment, (B) is also incorrect. (C) is not a valid justification because there is nothing to indicate that the seizure would fall under the "inevitable discovery" exception to the exclusionary rule.

The defendant, while visiting the victim, asked for permission to borrow the victim's car so he could drive to a convenience store to buy cigarettes. In fact, he intended to keep the car and sell it for cash. The victim agreed, and the defendant took the car and drove off. After thinking about it further, the defendant decided that it would be wrong to sell the victim's car, and headed back to the victim's house. On the way back, the car was destroyed in a collision through no fault of the defendant. May the defendant be convicted of larceny? A No, because he intended to return the car and therefore lacked the requisite mens rea for the crime. B No, because he abandoned his plan of selling the vehicle. C Yes, because withdrawal is not a defense to the crime. D Yes, because he intended to permanently deprive the victim of the car when he drove off in it.

D. The defendant has committed larceny. The defendant's change of heart after taking the car will not provide him with a defense because it is irrelevant. Larceny requires the taking and carrying away of the tangible personal property of another by trespass. In the instant case, the larceny was committed at the time he took the victim's car with the intent to permanently deprive him of possession. (Note that the taking was trespassory because the defendant obtained possession by misrepresentation—larceny by trick.) Given that the crime was complete once the defendant drove off, he is guilty of larceny, which makes (D) the correct answer. (A) is incorrect because the defendant did not intend to return the car at the time of the taking and thus had completed the crime at that time. He had the requisite mens rea at the time of the taking and carrying away, and this is sufficient to convict. (B) is incorrect because abandonment may be a defense in some states to an attempt of a crime. In the instant case, the defendant had already completed the crime, as described above. Thus, abandonment cannot act as a good defense. Choice (C) is incorrect because withdrawal is not relevant to the crime of larceny.

The defendant, angered because a rival gang member had twice beaten him up after school, obtained a heavy lead pipe and waited in a deserted alleyway which he knew the rival took as a route home every day after school. When his enemy came walking down the alley, the defendant leapt out behind him and smashed the pipe into the victim's head, knocking him to the ground. The defendant then rolled the victim over and pounded his face with 15 to 20 heavy blows with the lead pipe, killing him. The jurisdiction defines first degree murder as murder committed with premeditation and deliberation. All other murders are defined as second degree murders. Why will the defendant be convicted of first degree murder (as opposed to second degree murder)? A The relationship between the defendant and the victim requires that a finding of first degree murder be made. B The degree of causative relationship between the defendant's acts and the death of the victim renders it murder in the first degree. C The nature of the acts causing death distinguishes the defendant's action as first degree murder. D The defendant's mental state up to and including the moment of the attack determines that the act is first degree murder.

D. The defendant will be convicted of first degree murder because of his mental state up to and including the moment of the attack. The degree of murder under the statute is determined by the defendant's mental state--whether the killing was intentional and accomplished after premeditation and deliberation. Here, the defendant obtained a lead pipe and waited in a deserted alleyway for the victim, which shows premeditation and deliberation. The defendant's relationship with the victim and the manner of killing may have evidentiary significance with regard to the defendant's mental state, but do not themselves distinguish first from second degree murder. Thus, (A) and (C) are incorrect. The causal relationship between the defendant's act and the death of the victim may determine whether the act is murder, but once that analytical hurdle has been passed, it has no further significance as to the degree of murder. Therefore, (B) is incorrect.

The defendant was at a bar with a couple of friends when he spotted a man who had gotten the defendant's friend fired from a job several weeks ago. Since that time, the defendant had been verbally harassing the man and calling him names. This particular night, the defendant went over to the man's table and flirted with his girlfriend. The man was infuriated after having taken the defendant's abuse for so long, so he jumped up and attacked the defendant with a knife. The defendant could have easily run away, but instead grabbed the man and slammed him backwards. The man went crashing through the front window and was severely cut by the broken glass. He died before he could be taken to the hospital. The defendant will most likely be guilty of what crime? A Voluntary manslaughter. B Involuntary manslaughter. C Murder D None of the above.

D. The defendant would most likely be guilty of none of the listed crimes because the defense of self-defense makes his homicide excusable. A person may use deadly force in self-defense if (i) he is without fault, (ii) he is confronted with unlawful force, and (iii) he reasonably believes that he is threatened with imminent death or great bodily harm. In a majority of states, a person may use deadly force in self-defense even if this could be avoided by retreating. Here, the defendant's use of force was privileged because it reasonably appeared necessary to defend him from the man's unlawful attack, and the defendant had no duty to retreat under the majority view. Furthermore, the defendant can claim the privilege of self-defense even though his words triggered the fight—calling someone names would not be considered adequate provocation that would make the defendant the aggressor. Hence, because the defendant's use of force was privileged, he cannot be convicted of any of the listed crimes, making (A), (B), and (C) incorrect.

The defendant was arrested, given Miranda warnings, and charged with burglary. At the police station, he telephoned his mother and asked her to come to the station to post bail. Instead, his mother immediately called the family attorney. In the meantime, the police had begun questioning the defendant. Although he never told the police to stop the questioning, his answers were at first vague or clearly unresponsive. During the course of the questioning, the family attorney phoned the station and told the police that she had been hired to represent the defendant and would be there in half an hour. The police did not inform the defendant of the attorney's call. Ten minutes later, the defendant admitted to committing the burglary, and signed a statement to that effect prepared by the police. The attorney arrived a few minutes later and advised the defendant to remain silent, but he told her that he had already signed a confession. How should the court rule on the attorney's pretrial motion to exclude the confession as evidence at trial? A Grant the motion, because the police had a duty to inform the defendant that an attorney was coming to represent him. B Grant the motion, because the defendant has been deprived of his Sixth Amendment right to counsel. C Deny the motion, because the defendant's statement admitting the crime was voluntary. D Deny the motion, because the defendant waived his Miranda rights.

D. The defendant's confession should be admitted because he waived his Fifth Amendment privilege against compelled self-incrimination after receiving Miranda warnings. Miranda v. Arizona requires that a person in custody be informed of his right to remain silent and his right to the presence of an attorney during questioning. A suspect may subsequently waive his rights by making a confession, as long as the waiver was knowing and voluntary. In this case, the defendant received proper Miranda warnings, and there is no indication that he did not understand what his rights were. Although his answers during questioning were initially unresponsive, he never asked for an attorney or indicated that he wished to remain silent, and he voluntarily confessed after a relatively short period of interrogation. Hence, he validly waived his Miranda rights. (A) is incorrect because the police have no duty to inform the defendant that an attorney is attempting to see him. The defendant's ignorance of his attorney's efforts has no bearing on whether he made a knowing waiver of his Miranda rights. (B) is incorrect because the defendant's right to counsel was not violated. Although the defendant does have a separate Sixth Amendment right to counsel under Escobedo v. Illinois because he has already been arrested and charged with the crime, this right would only be violated if the defendant, after being informed of his right to counsel, had requested an attorney or had been prevented from seeing his attorney. Here, he made no request to see an attorney - even when he called his mother - and his attorney was allowed to see him immediately upon her arrival. Thus, he has waived his Sixth Amendment right to counsel. (C) is incorrect even though it is true that the defendant made a voluntary statement. Due process requires that for confessions to be admissible, they must be "voluntary," based on the totality of the circumstances, and here all of the circumstances indicate that the defendant's confession was voluntary. However, even a voluntary confession will be inadmissible if it was obtained in violation of Miranda rights. (D) is therefore a better choice than (C).

A felon intending to rob a market waited outside until there were no customers. When he saw that the market was empty, he went inside and walked up to the counter with his hand in his jacket pocket to simulate a gun. Before the clerk could turn around to see what the felon wanted, another customer entered the market, startling the felon, who turned and ran out the door. Should the felon be found guilty on a charge of attempted robbery? A No, because he used no actual force on the clerk nor threatened any. B No, because he withdrew successfully from the robbery attempt. C No, because he never entered the zone of perpetration. D Yes, regardless of whether he totally abandoned his plan when the customer entered the market.

D. The felon should be found guilty of attempted robbery. With the specific intent to commit a robbery, the felon went beyond mere preparation for the offense. Once a person has gone beyond preparation, abandonment is not a defense to attempt. A criminal attempt is an act that, although done with the intention of committing a crime, falls short of completing the crime. The defendant must have the intent to perform an act and obtain a result that, if achieved, would constitute a crime. Also, the defendant must have committed an act beyond mere preparation for the offense. If a defendant has, with the required intent, gone beyond preparation, the general rule is that abandonment is not a defense. Even in those jurisdictions in which abandonment is a defense, such abandonment must be: (i) fully voluntary and not made because of the difficulty of completing the crime or because of an increased risk of apprehension; and (ii) a complete abandonment of the plan made under circumstances manifesting a renunciation of criminal purpose, not just a decision to postpone committing it or to find another victim. Here, the felon intended to take money from the clerk at the market by means of the threat of having a gun (i.e., by simulating a gun). Thus, the felon intended to commit a robbery. In walking up to the market counter while simulating a gun with his hand, the felon committed an act that was a substantial step toward commission of the intended crime, and that strongly corroborated his intent and purpose to commit the crime. All that was missing to complete the crime was for the clerk to turn around and, upon seeing the felon apparently armed, be forced to give up the money. Thus, the felon went far beyond mere preparation for the crime of robbery. Having gone beyond mere preparation, with the intent to commit robbery, the felon is guilty of attempted robbery. And, as explained above, even if the felon abandoned his plan when the customer entered the market, such abandonment will not afford him a defense. Even in those jurisdictions in which abandonment is a defense, the felon will not have a defense because his abandonment apparently occurred when the customer's sudden presence increased the risk of apprehension. Thus, the abandonment was not fully voluntary and did not really manifest a renunciation of criminal purpose. (A) is incorrect because, to be guilty of attempted robbery, events need not have progressed to the point where the defendant has used or threatened to use force. Because the felon had the requisite intent for attempt and went beyond mere preparation by standing at the counter and simulating possession of a gun, he should be found guilty of attempted robbery. (B) is incorrect because, as detailed above, abandonment of an attempt does not afford a defense, and in any event, the felon's abandonment here was not fully voluntary because the felon abandoned his plan due to an increased risk of apprehension. (C) is incorrect because a conviction of attempt does not require entry into a "zone of perpetration." Rather, a defendant (with the requisite intent) need only have committed an act beyond mere preparation. The Model Penal Code and most state criminal codes require that the act constitute a substantial step toward commission of the crime and strongly corroborate the actor's criminal purpose.

Which of the following statements is true about the right to remain silent under Miranda? A A detainee may invoke the right to remain silent by not saying anything after being given Miranda warnings. B If the detainee invokes the right to remain silent, all questioning must cease unless and until the defendant reinitiates questioning. C The right to remain silent can be waived only in the presence of the detainee's attorney. D The police may reinitiate questioning if they scrupulously honor a request to remain silent.

D. The police may reinitiate questioning if they scrupulously honor a request to remain silent. The Supreme Court has allowed the police to reinitiate questioning where: (i) the police ceased questioning immediately upon the detainee's request and did not resume questioning for several hours; (ii) the detainee was given a fresh set of Miranda warnings; and (iii) the questioning was limited to a crime that was not the subject of the earlier questioning. It is not true that if a detainee invokes the right to remain silent, all questioning must cease unless and until the defendant reinitiates questioning. As discussed above, the police may reinitiate questioning about a different crime after several hours and rewarning the detainee. If a detainee is given Miranda warnings and says nothing, he has not invoked the right to remain silent. The right to remain silent must be explicitly invoked. If the detainee does not explicitly state that he wishes to remain silent, the police may continue to question the detainee. It is not true that the right to remain silent can be waived only in the presence of the detainee's attorney. While a detainee has a right to an attorney, he may waive the right and decide to talk on his own.

The privilege against self-incrimination is waived: A by a witness once she takes the stand, but a defendant must affirmatively waive the privilege B by a defendant or witness once she takes the stand C by a witness once she discloses incriminating information, but she cannot be compelled to reveal anything further D by a defendant when she takes the stand, but only to the extent that she is subject to cross-examination

D. The privilege against self-incrimination is waived by a defendant when she takes the witness stand, but only to the extent that she is subject to cross-examination. By taking the stand, the defendant waives the privilege to the extent necessary to subject her to any cross-examination proper under the rules of evidence. The choices indicating that a witness waives the privilege once she takes the stand are incorrect. A defendant's taking the stand constitutes a waiver because a defendant may refuse to take the stand completely. A witness, on the other hand, cannot refuse to take the stand, but instead must take the stand and assert the privilege, if applicable, to the specific questions posed. A witness will be held to have waived the privilege only to the extent that she discloses incriminating information. If a witness discloses incriminating information, she CAN be compelled to reveal further information as long as the disclosure does not increase her risk of conviction or create a risk of conviction for a different offense.

Under common law, the elements of a conspiracy include: A A written agreement between two or more persons, an intent to enter into an agreement, and an intent to achieve the objective of the agreement. B An agreement between two or more persons, an intent to enter into an agreement, an intent to achieve the objective of the agreement, and completion of the crime originally agreed upon. C An agreement between two or more persons, an intent to achieve the objective of the agreement, and completion of the crime originally agreed upon. D An agreement between two or more persons, an intent to enter into an agreement, and an intent to achieve the objective of the agreement.

D. Under common law, the elements of conspiracy include an agreement between two or more persons, an intent to enter into an agreement, and an intent to achieve the objective of the agreement. The agreement between the parties need not be written. Although most states require some overt act in furtherance of the conspiracy for the crime of conspiracy to be complete, the crime agreed upon does not need to be completed for the conspirators to be guilty of conspiracy.

found herself in immediate need of cash, and decided to burn down the cottage to collect the insurance on it. She waited until one evening when the tenant was away. The cottage owner then used her own key to gain access to it. To make it appear that the fire was caused accidentally by the tenant, she soaked one end of the mattress on the bed in the bedroom with gasoline and then left a lighted cigarette burning at the other end of the mattress. She planned that the cigarette would ignite the mattress and that when the fire smoldered to the area soaked in gasoline, the entire bed would burst into flames, and the resulting fire would destroy the house. However, the tenant returned home earlier than expected and discovered the fire just as the mattress burst into flames. He immediately put it out with a fire extinguisher. A police investigation revealed the cottage owner's activities. The cottage owner is guilty of: A Burglary as to the house and arson as to the mattress. B Neither burglary nor arson because she owned the structure and its contents. C Burglary and attempted arson. D Attempted arson but not burglary because she entered with her own key.

The cottage owner is guilty of burglary because the right of occupancy belonged to the tenant. However, the fact that there was no burning of the structure means that the cottage owner is guilty of attempted arson, rather than arson. Burglary at common law is a breaking and entering of the dwelling of another at nighttime, with the intent of committing a felony therein. A breaking requires some use of force to gain entry, but minimal force is sufficient. In determining whether the dwelling is that of another, occupancy rather than ownership is material. Thus, an owner can commit burglary of her own structure if it is rented and used as a dwelling by someone else. Here, although the cottage owner owned the cottage, the tenant had the right to occupy it pursuant to a lease. Thus, for purposes of the crime of burglary, the cottage owner is deemed to have entered the dwelling of another. Although the cottage owner used her own key to gain access to the cottage, this was still an unconsented use of force to effectuate entry, thereby constituting a breaking. This breaking and entering of the tenant's dwelling occurred in the evening. At the time of the entry, the cottage owner intended to commit the felony of arson. Consequently, all the elements of burglary are in place, making her guilty of this crime. Arson consists of the malicious burning of the dwelling of another. There is a requirement of some damage to the fiber of the wood or other combustible material. As with burglary, ownership of the structure is not material for determining whether the dwelling is that of another; rather, the right to occupancy is material. The cottage owner left a lighted cigarette on the mattress, intending to burn down the entire cottage. However, the tenant extinguished the fire before any damage was done to the structure of the cottage, even mere charring. Absent such damage, arson cannot have been committed. The cottage owner did commit attempted arson. A criminal attempt is an act which, although done with the intention of committing a crime, falls short of completing the crime. The defendant must intend to perform an act and obtain a result that, if achieved, would constitute a crime. Also, the defendant must have committed an act beyond mere preparation for the offense. The cottage owner intended to perform an act that would have culminated in the crime of arson. By soaking the mattress with gasoline and leaving a lighted cigarette on it, the cottage owner committed an act that came dangerously close to successfully burning the cottage. This act, in combination with the intent to commit arson, means that the cottage owner is guilty of attempted arson. (A) is incorrect because there can be no arson as to the mattress. Arson requires a burning of a dwelling. Because the cottage was not burned, the cottage owner is not guilty of arson. (B) is incorrect because the key element in determining whether a dwelling is that of another, for both arson and burglary, is the right of occupancy. Under the terms of his lease, the tenant had the right to occupy the cottage for one year. Therefore, the cottage owner's ownership of the cottage will not be a defense to either arson or burglary. (D) is incorrect because, as explained above, the cottage owner's use of a key to gain access to the cottage without the consent of the person who had the right of occupancy is deemed to be a use of force to gain entry, in the same way as if a person who did not own the cottage were to gain entry by means of a key.


Ensembles d'études connexes

5th grade Abeka History chapter 13 Check Up Section G

View Set

Medcerts Comptia A+ Practice Exam

View Set

Chapter 14: Massachusetts Licensing Law

View Set